Sie sind auf Seite 1von 312

Manual for the

GRE

Version 7.0

Copyright 2011 by The Princeton Review, Inc. All Rights Reserved.


Version 7.0
All rights reserved. No part of this manual may be reproduced for distribution to a third party in any
form or by any means, electronic or mechanical, including photocopy, recording, or any information retrieval system, without the prior express written consent of the publisher, The Princeton Review.
This manual is for the exclusive use of The Princeton Review course students, and is not legal for resale.
800-2Review
PrincetonReview.com

Acknowledgments
Extra special thanks to Joy Grieco, Neill Seltzer, Curtis Retherford,
Peter Hanink, Brian Singer, and Neil Thornton.
Special thanks to the following for their many contributions to this manual:
Andrew Brody, Heather Brady, Jennifer Downey, Kim Howie, Liz Rutzel,
Meave Shelton, and the staff and students of The Princeton Review.
The Princeton Review would like to acknowledge the question authors and
quality control experts without whose invaluable work this manual and course
would not have been possible:
Question authors:
Jennifer Amerkhanov, Stephanie Aylward, Brian Becker, Kevin Cook,
Kirsten Frank, Mark Hellman, Jay Hilsenbeck, Beth Hollingsworth,
Karen Hoover, Melissa Janae, Paul Kugelmass, Michael Levy, Aaron Lindh,
Eliz Markowitz, Lisa Mayo, Amy Minster, Joshua Morris, Jerome J. ONeill,
Elizabeth Owens, Henry Price, Anthony Pumilia, Debbi Reynolds,
Tim Ricchuiti, Lisa Rothstein, Audra Rouse, Janet Stolzer, Emily Swenson,
Scott Thompson
Quality control specialists:
Michael Chapian, Jennifer Downey, Karen Hoover, Rodi Steinig

Table of Contents
Introduction ..........................................................................

Lesson 1 Math ......................................................................

15

Lesson 2 Math ......................................................................

35

Lesson 2 Verbal .....................................................................

49

Lesson 3 Math ......................................................................

59

Lesson 3 Verbal .....................................................................

77

Lesson 4 Math ......................................................................

95

Lesson 4 Verbal .....................................................................

127

Lesson 5 Math ......................................................................

139

Lesson 5 Verbal .....................................................................

159

Lesson 6 Math ......................................................................

177

Lesson 6 Verbal .....................................................................

201

Lesson 7 Math ......................................................................

217

Lesson 7 Verbal .....................................................................

237

Lesson 8 Math ......................................................................

243

Lesson 8 Essays ....................................................................

259

Verbal Practice ......................................................................

267

Math Practice .......................................................................

281

Between Now and Test Day ..................................................... 297

Introduction

MANUAL FOR THE GRE

DO YOUR RESEARCH
You cant look at a GRE score in a vacuum; your score is just a number. In order to know how much work
you need to put in over the next several weeks, you need to know your starting score, your target score, and
the role of the GRE in the admissions process at your target programs. Here are some questions you should
be asking of your desired programs:
1.
2.
3.
4.
5.
6.
7.

How important are scores?


What else is required for admissions?
What do you do with multiple scores?
Are you looking at all parts of the score?
Will scores be used for anything else?
How competitive is admissions?
What was the average GRE score for last years incoming class?

Admissions will ask you two questions that you should be asking yourself:
Why this program and why now?

The Princeton Review, Inc.

INTRODUCTION

JUST WHAT IS THE GRE?


The GRE (Graduate Record Examination) is a standardized, multiple-choice test that is supposed to assess
your ability in three general areas: math skills, verbal skills, and writing skills. The test is required by most
university graduate programs. When considering you as a candidate, these programs weigh your GRE score
in addition to your academic history, recommendations, and personal essays. Some programs also use the
GRE as a determinant when awarding fellowships and grants.

HOW IMPORTANT IS THE GRE?


Unfortunately, theres no straightforward answer to this question. Some graduate programs consider the
GRE very important; others consider it a mere formality. Still other programs do not use the GRE in the
admissions process, though they use it when awarding financial aid. Also, different departments look at
different parts of the test. For example, if you are considering enrolling in a graduate program in English
literature, the quantitative portion of the GRE may not matter at all to your prospective schools. Similarly,
a program in applied mathematics may consider the verbal portion immaterial. Some programs will not
care how you performed on individual sections but will ask for a minimum composite score (made up of
your performance on all parts of the GRE). If youd like more specifics, contact the schools in which youre
interested. Speak directly with someone in your prospective graduate department. Department secretaries
and officers can often tell you precisely how their department will use your GRE scores when considering
your application.

The Princeton Review, Inc.

MANUAL FOR THE GRE

WHAT DOES THE GRE TEST?


The Analytical Writing Assessment (AWA) section probably comes the closest to measuring what it actually
purports to measurenamely your ability to support an opinion and to critically evaluate arguments made
by others.
The test author, Educational Testing Service (ETS), claims that the GRE measures certain developed verbal, quantitative (math), and analytic abilities that are important in academic achievement. Okay. But
what does that actually mean?
If youve already taken the GRE, you know that it covers such basic math skills as algebra and geometry,
such writing skills as formulating and critiquing arguments, and such verbal skills as reading comprehension
and vocabulary. By testing your abilities in these areas, ETS argues that the test necessarily reflects the
opportunities and efforts that have contributed to those abilities.
Of course, thats what ETS would say. ETS has a vested interest in maintaining its monopoly on this and
other standardized tests. Whatever the GRE purports to measure, it does not test the skills you developed
in college, nor is it in any way an intelligence test. Most important of all and regardless of ETSs claims, the
GRE has nothing to do with aptitude for graduate study. In fact, it has never been demonstrated that there
is any correlation between performance on the GRE and ability to tackle graduate work in any field. The
bottom line is quite simple:

The GRE tests how well you take the GRE.

Why, then, do you have to take the GRE, and why do schools use it when considering your candidacy? The
answer is simple. Given our different undergraduate backgrounds, schools like having a way to compare
everyone by a single measure.

The Princeton Review, Inc.

INTRODUCTION

WHO IS ETS?
As you may already know, ETSthe same folks who ruined your high school years with the PSAT, SAT,
and SAT Subject Testsis responsible for the GRE.
ETS writes the other exams for graduate study, including the GRE Subject Tests, as well as exams for CIA
agents, barbers, golf pros, and travel agents. ETS is a private, nonprofit corporation (though it does have
highly profitable for-profit divisions). It is not supervised by the government. It is not supervised by anyone,
at any level. What gives ETS the right to administer this test? The fact that it gives this test. To summarize:

ETS has the right to administer the GRE, which tests how well
you take the GRE, because it administers the GRE.

The Princeton Review, Inc.

MANUAL FOR THE GRE

HOW DOES ETS WRITE THE TEST?


ETSthe Big Brother of standardized testingknows how you think. Through extensive testing of individual problems, and by taking advantage of the ways in which we take standardized tests, ETS ensures that
the GRE always produces the same results.

Unpaid Guinea Pigs


On nearly every test ETS administers there are experimental questions. These questions do not count toward
your score. They are used by ETS to ensure that the questions that ultimately appear on real tests produce
the results it desires. Any question that fails to do so is promptly rewritten or thrown out.
As unfair as it may be for ETS to have you pay it in order to do its research and development work, you
dont have any choice in the matter. The experimental questions are not optional. This is one way in which
ETS guarantees that its tests produce perfect curves. Another way is by taking advantage of our test-taking
tendencies.

Setting Traps
ETS is remarkably good at setting traps for the average test taker. For example, the worst thing you can do
on the GRE is spend too much time on hard questions and rush through easy ones. Yet ETS makes it seem
as if the only way to do well on the test is by putting the same amount of time into every question.
Naturally, theres more to the GRE than simply a series of traps. However, even when dealing with a
problem that seems quite easy, you may unwittingly stumble into an ETS pitfall. Utilizing the strategic
techniques and comprehensive review covered in this course, youll avoid the GREs many traps and beat
ETS at its own game.
Now that you know a little about ETS, take a look at the GRE.

The Princeton Review, Inc.

INTRODUCTION

ELEMENTS OF THE TEST


Section

Number of Questions

Time

Analytical Writing

One Issue essay and one


Argument essay

30 minutes per essay


1 hour total
No breaks between essays

Verbal Reasoning

Two 20-question sections

30 minutes per section

Quantitative Reasoning

Two 20-question sections

35 minutes per section

Experimental

One 20-question section

30/35 minutes

Research

Varies

Varies

The Verbal, Quantitative, and experimental sections can occur in any order. ETS says that it reserves the
right to slightly alter the number of questions.

The Essays
The first scored section of your test will be the two essay tasks. You will have a one-minute break after this
section but no break between the essays.

Verbal and Quantitative


You will have two Verbal and two Quantitative sections. They could come in any order. After the essays and
the subsequent two multiple-choice sections, you will have a 10-minute break. There is a one-minute break
after each of the other sections.

Experimental
You will actually see three Quantitative sections and two Verbal sections, or three Verbal and two Quantitative. The extra Quantitative or Verbal is experimental. The experimental section does not count towards
your score. Other than knowing that it is Quantitative or Verbal, there is no way to figure out which section
is experimental, so you will have to take each section seriously.

The Princeton Review, Inc.

MANUAL FOR THE GRE

How Adaptive by Section Works


The GRE is adaptive by section. You will see at least two Quantitative and two Verbal sections. The more
questions you get right on the first section, the harder the questions are that you will see on the second section. You have to do well on both sections to hit your target score.

ELEMENTS OF THE COURSE


1.
2.
3.

4.

5.

ClassClasses will cover test skills, homework and drill questions, test review, and difficult
test content.
TestsThe course includes five full-length, online, GRE practice tests that simulate the actual
exam.
Test ReviewA half hour before and after every class is reserved for one-on-one test review.
This is to be scheduled with your teacher and is available only to students who have completed
all scheduled tests and drills.
Homework DrillsOn your online student center you will find a series of drills designed to
reinforce key GRE skills. The first portion of each class is reserved for homework review. Each
drill question has a Review in Class button so that any and all of your content questions will
get covered in class.
Online LessonsSome GRE concepts will be introduced in online lessons. These concepts
will be revisited and reinforced in class. Students who are already comfortable with a given
concept can test out of a given lesson. Students who need more help with a concept will have
access to extended practice drills and can also mark questions for review in class.

The Princeton Review, Inc.

INTRODUCTION

SCORE IMPROVEMENTS
Score Improvement Expectations
Class Only
Class & Some Tests
Class & All Tests
Class, Tests, & Some Drills
Class, Tests & All Drills

Results Come in Stages


Taking the GRE is a skill and, like any skill, it requires practice. You might understand how to play the piano
the first time you sit down to play one, but that doesnt mean you will be good at it. Mastering the piano
requires long hours of practice. Mastering the GRE does too. The good news is that your score is entirely in
your hands.

The Princeton Review, Inc.

MANUAL FOR THE GRE

TECHNIQUES
All of the questions on the test can be divided into two categories:
Category 1: The Questions You Are Supposed to Get Right. These are the questions that involve math you are

comfortable with or vocabulary words you know.


Category 2: The Questions You Are NOT Supposed to Get Right. When the folks at ETS want you to get a

question wrong, they will find a way. On the Verbal section, hard questions include arcane vocabulary
words youre not supposed to know; on the Math section, hard questions include wrong but tempting
answer choices that you are supposed to pick.

The techniques do three jobs, all equally important.


First, they ensure that you answer correctly the questions that
you should get right. Second, they make hard questions easier.
Third, they help you manage your time more efciently.

10

The Princeton Review, Inc.

INTRODUCTION

Global Technique #1: Take the Easy Test First


Within a section, each question counts equally towards your score. However, some questions will be easy
and others will be difficult. The beauty of the GRE is that you can answer questions in any order you like.
A question you can nail in 25 seconds is worth just as much as a tough question. To maximize your score,
leave the questions you dont like for last. If you are going to run out of time (and, unless you are shooting
for a 160 or higher, you should be running out of time), make sure the questions you end up bubbling in are
the ones you didnt want to work on anyway. We will practice this extensively throughout the course, but
remember: Skip early and skip often.

Global Technique #2: Scratch Paper


One of the genuinely helpful tools ETS gives you on the GRE is scratch paper. Over a fourhour test, your
brain is going to get tired. Keeping your hand moving is a way to stay focused on the task at hand. If your
brain is communicating with your hand, then your brain is engaged rather than preoccupied with reading
the question three times in a row, thinking about what youre going to do when the test is over, or any other
random things. You wont outthink ETS, but you can out-process them.
For each different question type you will learn a graphic set-up to organize information and answer all
questions as efficiently and accurately as possible.

Put your set-ups in the


upper left corner.
Clearly number each problem so that
you can find your work if you return
to the problem.
When youre done with each
problem, draw a line underneath it,
across the page, so that you have a
clean space upon which to work the
next problem.

1.

A a b c d B
65 x = 1 y = 1 34
34 x = 2 y = 1 65
34 x = 2 y = 1 65
8

2.
2

2
2

x
y
2

3.

Note how every answer choice has


been checked and every problem has
its own distinct space.

Reserve the right side for


scratch work.

A
B
C
D
EE

10

32
2
64

x=4
y=6
56

7
80
- 24
56

1030
5
32

30

A
B
C
D
EE

8 + 12 + 4
8 + 12 + 16
16 + 24 + 8
16 + 24 + 8

250
30

16
24
8
48

32
24
56

32

25 10
16 2
55 2 5
442
2 2 2 2 2=2 5

4.

The Princeton Review, Inc.

11

MANUAL FOR THE GRE

Questions 17 are Text Completions.


1 Blank
text completions.
Note that this student
has come up with her
own words for the
blank and has considered
and marked every
answer choice.
Marked questions
are clearly marked on
scratch paper.
Questions 811/12 are
Reading Comprehension.
Each passage has been
mapped. Each
answer choice has been
considered.
Questions 1215 or
1316 are Sentence
Equivalence. Note that
this student has come up
with her own words for
every blank and considered every answer choice.

reflects
1. A
B
C
D
E

3 Blank
text completions.

2 Blank
text completions.

obvious
3. A
x
B
C
x

false
2. A
B
C
D
E

wrong
x
x

The Princeton Review, Inc.

x
x

latch
portray onto
5. A
x
B
x
x
C
x

fakes increased trend?


6. A
B
C

x
x

x
x

7. A
B
C
D
E

x
x

Prob.
1. Sediment: Climate Hist.
2. Climate affects sediment
3.
4. Cycle peaks at 1300 & 2400
care
12. A
B
C
D
E
F

common
13. A
B
C
D
E
F

taciturn
14. A
B
C
D
E
F

8. A
B
C
D
E

9. A
B
C
D
E

10. A
B
C
D
E

m
11 A
B
C
D
E

lying
15. A
B
C
D
E
F

Every question is clearly numbered


so that the student can easily return
to the question if needed.

12

4. A
B
C

x
x

There is appropriate space


between each column of answer
choices to avoid crowding and
to leave room for notes and marks.

INTRODUCTION

Global Technique #3: The Mark Button


Reading or calculation errors on a four-hour test are unavoidable. The problem is that a misread question or
a calculation error will completely change the way you see the problem, and once you see a question wrong,
it is almost impossible to see it correctly. As long as you stay with a misread question, you will continue to
see it wrong. Meanwhile, the clock is ticking and youre not getting any closer to the answer. We call this
Quicksand. Once youre in Quicksand, it is very difficult to get out.
On the flip side, once youve spotted the error, solving the problem correctly is often a straightforward
process. A question that bedeviled you for minutes on end may appear to be appallingly obvious later. The
trick is to change the way you see the question while you still have the opportunity to fix it.
Here are a few signs that you are in Quicksand:

Youve found an answer, but it is not one of the choices theyve given you.
You have half a page of calculations but are no closer to an answer.
Youve spent more than four minutes on a problem.
Your hand is not moving.
Youre down to two answer choices and both seem correct.
Youve eliminated all of the answers.
There is smoke coming out of your ears.
Youre beginning to wonder if ETS made a mistake.

If you find yourself in any of these situations, you are in Quicksand. Stop what youre doing and get out.

Step 1 Recognize you are in Quicksand.


Step 2 Mark and move.
Step 3 Distract your brain by doing two or three other questions.
Step 4 Return to the problem and take a second look.
Ways to see the problem with fresh eyes:

Use your finger on the screen to force yourself to read the problem word for word.
Ask yourself if there are different ways to express the information.
Can you use the answer choices to help?
Can you paraphrase the answer choices?
If the path to the right answer is not clear on a second viewing, guess and walk away again.
Why stick with a problem you dont know how to solve?

The Princeton Review, Inc.

13

MANUAL FOR THE GRE

Global Technique #4: Pacing


Speed kills on the GRE. The clock has a way of infecting your brain. Take a section untimed, and in
addition to answering more questions, youll make fewer mistakes. The questions dont get any harder when
there is a clock, yet somehow most testers get more wrong. To make matters worse, the questions you get
wrong are likely to have taken you far more time than the ones you got right.
The trick is to take each section as if there is no clock. As long as you are skipping the hard ones and skipping and coming back when you run into resistance on questions youve started, you should get very few
questions in a section wrong.
Remember that it is not the number of questions that you answer that gives you your score, it is the number
of questions you answer correctly. Accuracy is everything. Ignore the clock. Slow down and work for
accuracy only. If you run into a brick wall, dont continue to spend time on the problem; go do an easier one
and come back. The minute you try to go faster, however, your accuracy will go down and your score along
with it.
There is only one exception to this, and that is the last two minutes of a section. A skipped question and a
wrong answer count the same. In other words, there is no penalty for guessing on a question you dont
know. When two minutes remain on your clock, stop what youre doing and bubble in answers to any
remaining unanswered questions. A few lucky guesses will pay off. If you dont get any of them right, no
harm done.

14

The Princeton Review, Inc.

Lesson 1
Math

MANUAL FOR THE GRE

1
Question 1
c=2
d=5
Quantity A
(d 4c)6

Quantity B
(d 4c)7

Quantity A is greater.
Quantity B is greater.
The two quantities are equal.
The relationship cannot be determined from the
information given.

Question 2
32
m
Quantity A
m

m
2
Quantity B
8

Quantity A is greater.
Quantity B is greater.
The two quantities are equal.
The relationship cannot be determined from the
information given.

Question 3
A

On line C, the length of BC is 1.5 times the length of


AB. If the length of AC is 30, what is the length of
BC ?
10

16

The Princeton Review, Inc.

12

15

18

20

LESSON 1 MATH

SCRATCH PAPER

The Princeton Review, Inc.

17

MANUAL FOR THE GRE

1
Question 4
Four containers of our are on the table:
1
The rst contains
of a pound, the second
3
1
1
contains
of a pound, the third contains
of a
6
9
1
pound, and the fourth contains
of a pound. If
18
each container can hold one pound of our, how
many additional pounds of our are required to ll
all four containers?
2

11

25

10

Question 5
Machine

Units Made per Hour

Percentage of Defective Units

2,800

12%

1,500

7%

750

6%

What is the ratio of the number of defective units


created in an hour by machine A to the number of
defective units created in an hour by machine B?

Question 6

n=

2
7

1
3

4
9

2
3

5
9

5
7

+ x

In the equation above, if n is an integer, which of the


following could be a possible value of x ?
Indicate all such values.

. 0
.

2
63

. 1
.

65
63

. 4
18

The Princeton Review, Inc.

LESSON 1 MATH

SCRATCH PAPER

The Princeton Review, Inc.

19

MANUAL FOR THE GRE

1
PLUGGING IN
Question 1
Doug is 3 times as old as Neill and half as old as Liz.
If Doug is d years old, what is the sum of their ages?
d
5
3
d
7
3
d
10
3
d
7
2

TRIGGER

d
9
2

Trigger: _______________________________________________________________

Question 2
If f =

, where g is not equal to 0 and f is not equal


g
g3
to 1, then which of the following is equal to
?
f 1
g

20

The Princeton Review, Inc.

gf

LESSON 1 MATH

SCRATCH PAPER

1.
A
B
C
D
E

d=
n=
l =

2. Set up your scratch paper.


3. Assign an easy number (e.g. 2, 5, 10, 100) to one variable.
4. Work through the problem.
5. Find the answer to the question. Thats your target number. Circle it.
6. Check all answer choices.

RESPONSE

1. Recognize the opportunity: PLUG IN!

2.
A
B
C
D
E

f=
g=

The Princeton Review, Inc.

21

MANUAL FOR THE GRE

1
Question 3
If a factory produces 1,500 light bulbs in one minute,
how many light bulbs will it produce in t seconds?
1,500t

25

25t

750

90,000

Question 4
For y 0 and y 1, which of the following represents
1
the reciprocal of

1?
y

y 1

y2 + 1

y +1

y 1

y +1

Question 5
A group of 10 people decides to share equally in an
apartment that costs r dollars to rent each month.
If x people drop out of the group, how much more,
in dollars, must each remaining person pay?
rx
10(10 x)
10r
x
r
10(10 x)
r
10 x
rx
10 x

22

The Princeton Review, Inc.

LESSON 1 MATH

SCRATCH PAPER

Check your work. For every problem, you should have terms labeled, a target number
circled, and all answer choices checked.

The Princeton Review, Inc.

23

MANUAL FOR THE GRE

1
Quant Comps
Question 6
y0
Quantity A
10y

Quantity B
y

 Quantity A is greater.
Quantity B is greater.

 The two quantities are equal.


 The relationship cannot be determined from the

TRIGGER

information given.

Trigger: _______________________________________________________________

Need a weird number? Try FROZEN:


F Fractions
R Repeats
O One
Z Zero
E Extremes
N Negative

Question 7
x>y>0
Quantity A
6x





24

The Princeton Review, Inc.

Quantity B
7y

Quantity A is greater.
Quantity B is greater.
The two quantities are equal.
The relationship cannot be determined from the
information given.

LESSON 1 MATH

SCRATCH PAPER

6.

y=
y=
y=

2. Draw your set-up.


3. Plug in an easy number (according to the problems rules).
4. Cross off answer choices.
5. Repeat using FROZEN.

7.

a b
x=

c d
y=

x=

y=

x=

y=

RESPONSE

1. Recognize the Opportunity: PLUG IN!

The Princeton Review, Inc.

25

MANUAL FOR THE GRE

1
Question 8
3 < x < 6 < y < 10
Quantity A
The greatest possible
value of y x






Quantity B
7

Quantity A is greater.
Quantity B is greater.
The two quantities are equal.
The relationship cannot be determined from the
information given.

Question 9
a, b, and c are consecutive even integers such that
a < b < c.
Quantity A
a+c






Quantity B
2b + 2

Quantity A is greater.
Quantity B is greater.
The two quantities are equal.
The relationship cannot be determined from the
information given.

Question 10
a0
Quantity A
|a 1|






26

The Princeton Review, Inc.

Quantity B
|a| 1

Quantity A is greater.
Quantity B is greater.
The two quantities are equal.
The relationship cannot be determined from the
information given.

LESSON 1 MATH

SCRATCH PAPER

The Princeton Review, Inc.

27

MANUAL FOR THE GRE

1
Must Be
Question 11
If a, b, and c are odd integers, which of the following
must also be odd?

TRIGGER

(a + b)c
ac + b
(a + b) (b + c)
abc
(b a) + (c b)

Trigger: _______________________________________________________________

Question 12
If p and q are integers, such that p < 0 < q, which of
the following must be true?
Indicate all such statements.

.
.
.

28

The Princeton Review, Inc.

2p < 2q
p2 < q2
p+q=0

LESSON 1 MATH

SCRATCH PAPER

11.
a=
b=
c=

a=
b=
c=

a=
b=
c=

A
B
C
D
E

2. Draw your set-up.


3. Plug in an easy number (according to the problems rules).
4. Cross off answer choices.
5. Repeat using FROZEN.

RESPONSE

1. Recognize the Opportunity: PLUG IN!

The Princeton Review, Inc.

29

MANUAL FOR THE GRE

1
DRILL
Question 1 of 7
Bill is twice as old as Heidi and six years younger
than Mel. If Heidi is h years old, how old is Mel in
terms of h ?

h4
h+4
 2h 4
 2h
 2h + 6

Question 2 of 7
Quantity A
6+x






Quantity B
6x

Quantity A is greater.
Quantity B is greater.
The two quantities are equal.
The relationship cannot be determined from the
information given.

Question 3 of 7
0 < x < 10
0<y<1
Quantity A
xy






30

The Princeton Review, Inc.

Quantity B
9

Quantity A is greater.
Quantity B is greater.
The two quantities are equal.
The relationship cannot be determined from the
information given.

LESSON 1 MATH

SCRATCH PAPER

The Princeton Review, Inc.

31

MANUAL FOR THE GRE

1
Question 4 of 7
Eleven years ago, Lauren was half as old as Mike
will be in 4 years. If Mike is m years old now, how
old is Lauren now in terms of m ?
4m 11

1
2

(m + 4) + 11

(m 11)

4m +

Question 5 of 7
P<Q
Quantity A

Quantity B

QP

Q P






3
Quantity A is greater.
Quantity B is greater.
The two quantities are equal.
The relationship cannot be determined from the
information given.

Question 6 of 7
If the sum of three consecutive odd integers is k,
then, in terms of k, what is the greatest of the three
integers?
k6

k3

k+3

k+6

Question 7 of 7
Item F costs three times as much as item G, and
item H costs $4 more than one-third the price of
item G.
Quantity A
The cost of item F






32

The Princeton Review, Inc.

11

Quantity B
The cost of item H

Quantity A is greater.
Quantity B is greater.
The two quantities are equal.
The relationship cannot be determined from the
information given.

2m 7

LESSON 1 MATH

SCRATCH PAPER

The Princeton Review, Inc.

33

MANUAL FOR THE GRE

2. Replace variable with number.


Variables in the answer
choices

3. Work the problem.


4. Identify and circle target number.

RESPONSE

TRIGGER

1. List A,B,C,D,E on scratch paper.

TRIGGER
|

The Princeton Review, Inc.

Quant Comp with variables

2. Plug in an easy number.


3. Eliminate two answer choices.
4. Repeat using FROZEN.

1. Draw set-up.
Must be

2. Plug in an easy number.


3. Eliminate answer choices.
4. Repeat using FROZEN.

RESPONSE

34

1. Draw set-up.

RESPONSE

TRIGGER

5. Check all answer choices.

Lesson 2 Math

MANUAL FOR THE GRE

EXPONENTS & ROOTS


Question 1
12
20

5
9
25
3 21
5 20
1
3
5
3 3

TRIGGER

5 5

Trigger: _______________________________________________________________

Question 2
x5 + x3
x 4 + x2

4x2

36

The Princeton Review, Inc.

x2

2x

x2

LESSON 2 MATH

SCRATCH PAPER
2

Convert large bases to their prime factors.

RESPONSE

Find and cancel common factors.

The Princeton Review, Inc.

37

MANUAL FOR THE GRE

Question 3
Quantity A

Quantity B

4 4
12

11

48

643






Quantity A is greater.
Quantity B is greater.
The two quantities are equal.
The relationship cannot be determined from the
information given.

Question 4
126 = 3a2b. What is the value of a + b ?

Question 5
Which of the following statements must be true?
Indicate all such statements.

.
.
.

38

The Princeton Review, Inc.

( x )3 = x 1
(x2) = ( x)2
x-2 < x2

LESSON 2 MATH

SCRATCH PAPER
2

The Princeton Review, Inc.

39

MANUAL FOR THE GRE

HIDDEN PLUG INS


Question 1
1
of her monthly income for food,
Sarah pays
8
1
1
4
for utilities,
for student loans, and
of the
8
8
5
remainder for rent. If at the end of each month
Sarah puts

of her remaining income into a CD


2
account, what portion of Sarahs monthly income

TRIGGER

does she put into the account?

10

80

16

20

Trigger: _______________________________________________________________

Question 2
In a certain apartment building, 40 percent of the
units have one bedroom, and the remaining units
have two bedrooms. If 20 percent of the onebedrooms and 10 percent of the two-bedrooms are
vacant, what percent of the units in the building are
vacant?
10%

40

The Princeton Review, Inc.

14%

15%

30%

40%

LESSON 2 MATH

SCRATCH PAPER
2

RESPONSE

Plug In for the unknown value or amount.

The Princeton Review, Inc.

41

MANUAL FOR THE GRE

PITA
Question 1
Vicken, Roger, and Adam went to buy a $90 radio. If
Roger agrees to pay twice as much as Adam, and
Vicken agrees to pay three times as much as Roger,
how much must Roger pay?

TRIGGER

$10

$20

$30

$45

$65

Trigger: _______________________________________________________________

Question 2
Mike bought a used car and had it repainted. If the
cost of the paint job was one-fth of the purchase
price of the car, and if the cost of the car and the
paint job combined was $4,800, then what was the
purchase price of the car?
$800
$960
$3,840
$4,000
$4,250

Question 3
Gerald is three times as old as his cousin Lucy and
14 years older than his parrot Polly. In 4 years Lucy
will be half as old as Polly will be then. How old is
Gerald?
24
42

The Princeton Review, Inc.

36

45

54

60

LESSON 2 MATH

SCRATCH PAPER
2
1.
A
B
C
D
E

R
10
20
30
45
65

2. List answer choices on your scratch paper.


3. Label the rst column.
4. Plug In (C).
5. Work the problem in bite-sized pieces, making a new column for each new step.
6. POE.

RESPONSE

1. Recognize the Opportunity: Plug In the Answers (PITA)!

The Princeton Review, Inc.

43

MANUAL FOR THE GRE

Plugging In Drill
Question 1 of 5
One-half the members of a team are juniors, onethird are sophomores, and the remainder are
seniors.
Quantity A
The number of juniors
on the team






Quantity B
The number of seniors
on the team

Quantity A is greater.
Quantity B is greater.
The two quantities are equal.
The relationship cannot be determined from the
information given.

Question 2 of 5
If 1 < x < 0, which of the following has the
greatest value?
1

x3

x
1

1+x

Question 3 of 5
In a high school pep band that consists of forty
students, the number of seniors is ve fewer
than twice the number of juniors, and 30% of the
students in the band are neither juniors nor seniors.
Quantity A
The number of juniors in
the band






44

The Princeton Review, Inc.

Quantity B
11

Quantity A is greater.
Quantity B is greater.
The two quantities are equal.
The relationship cannot be determined from the
information given.

LESSON 2 MATH

SCRATCH PAPER
2

The Princeton Review, Inc.

45

MANUAL FOR THE GRE

Question 4 of 5
S is a sequence s1, s2, s3sn in which every term
after the rst is one less than three times the
previous term. If s5 s3 = 28, which of the following
is the rst term in the sequence?
2
3
8
9
1
5
3
2

Question 5 of 5
If 20 percent of the trees in a certain park are
evergreens, and 40 percent of the non-evergreens
are maple trees, and there are 75 percent as many
oak trees as maple trees in the park, what fraction
of the trees in the park are not maples, oaks, or
evergreens?
1
10
3
25
1
5
6
25
1
4

46

The Princeton Review, Inc.

LESSON 2 MATH

SCRATCH PAPER
2

The Princeton Review, Inc.

47

MANUAL FOR THE GRE

TRIGGER

Exponents

Find and cancel common factors.

RESPONSE

TRIGGER

SUMMARY

Numbers too big to calculate

Convert large bases to their prime


numbers.

RESPONSE

TRIGGER
|

1. List answers on your scratch paper.


2. Label the rst column.

or

3. Assume (C) to be correct.

An urge to write your own algebraic


formula

4. Use (C) to work the problem.


5. POE.

Plug In for the unknown value or amount:


Answer choices expressed as fractions or
percentages

The Princeton Review, Inc.

If fractions, plug in common


denominator.
If percentages, plug in 100 for total.

RESPONSE

48

The phrases how much, how many,


what is the value of

RESPONSE

TRIGGER

Lesson 2 Verbal

MANUAL FOR THE GRE

Section A

Reading Comp.

Text Completion

Text Completion

Sent. Equivalence Reading Comp.

10 11 12 13 14 15 16 17 18 19 20

Reading Comp.

Sent. Equivalence Reading Comp.

Section B

Although the Math sections test the same rules over and over again, the Verbal sections have passages from
many different disciplines and an incredible variety of vocab. The time required to complete verbal problems can also vary much more than the time required for math problems. So, as important as they are on
the Math sections, proper pacing and POE are paramount on the Verbal sections.

Dont sacrice accuracy for speed:


Get more efficient at Text Completions and Sentence Equivalence so that you have more
time for Reading Comprehension.
Four NOs = a YES:
Find the wrong answers.

Using the Mark Button


The Mark button is particularly important on the verbal side of the test. As your brain gets tired, you are
more likely to misread questions or passages. Once this happens, no matter how many times you reread it,
you will continue to misinterpret it in the exact same way. Click the Mark button and move on, and return
after several questions.

When taking the GRE, the inexperienced test taker will take the
the test in the order in which it is given, ignoring the level of
difculty, the number of questions, and the ability to change the
order in which the questions are answered.

50

The Princeton Review, Inc.

LESSON 2 VERBAL

SCRATCH PAPER
All Verbal scratch paper looks the same. It looks like this:

Proper pacing and efficient use of POE require effective use of ones scratch paper. For instance, for Reading
Comprehension questions, youll write out A, B, C, D, and E on your scratch paper for each problem, so
you can mark each answer choice. Use a two-pass approach to POE so that you do not get bogged down
on any one answer choice: 1) Eliminate the answer choices that are clearly wrong. 2) Take your second pass
through the remaining choices using the strategies youll learn in the upcoming lessons.
Use these symbols:

When you are done, your scratch paper will look something like this:

The Princeton Review, Inc.

51

MANUAL FOR THE GRE

2
Line
5

10

15

20

25

30

35

52

Until recently, corporate ideology in the United


States has held that bigger is better. This traditional view of the primacy of big, centralized
companies is now being challenged as some of
the giants of American business are being outperformed by a new generation of smaller, streamlined businesses. If it was the industrial revolution
that spawned the era of massive industrialized
companies, then perhaps it is the information
revolution of the 1990s that is spawning the era
of the small company.
For most of this century, big companies dominated an American business scene that seemed
to thrive on its own grandness of scale. The
expansion westward, the growth of the railroad
and steel industries, an almost limitless supply
of cheap raw materials, plus a population boom
that provided an ever-increasing demand for new
products (although not a cheap source of labor)
all coincided to encourage the growth of large
companies.
But rapid developments in the marketplace
have begun to change the accepted rules of
business and have underscored the need for fast
reaction times. Small companies, which lack huge
overhead and inventory, can respond quickly to a
technologically advanced age in which new products and technologies can become outmoded
within a year of their being brought to market.
Of course, successful emerging small companies face a potential dilemma in that their very
success will tend to turn them into copies of the
large corporate dinosaurs they are now supplanting. To avoid this trap, small companies may look
to the example of several CEOs of large corporations who have broken down their sprawling
organizations into small, semi-independent
divisions capable of making their way into the
twenty-rst century.

The Princeton Review, Inc.

Question 1
The primary purpose of the passage is to
present evidence that resolves a
contradiction in business theory
discuss reasons an accepted business
pattern is changing
describe a theoretical model and a method
whereby that model can be tested
argue that a traditional ideology deserves
new attention
resolve two conicting explanations for a
phenomenon
Question 2
Which of the following best describes the
organization of the passage?
A conventional model is described and an
alternative is introduced and evaluated.
An assertion is made and a general
supporting example is given.
Two contradictory points of view are
presented and evaluated.
A historical overview is given to explain a
phenomenon.
An organizational trend is described and
then criticized.

LESSON 2 VERBAL

SCRATCH PAPER
Genre (What)

Problem
Question
Conflict
Paradox
Change
Innovation
Discovery

Purpose (Why)

Predict
Recommend
Inform/Explain
Correct
Evaluate

Structure (How)

Cause/Effect
Chronology
Classification
Comparison/Contrast
Steps/Stages

The Princeton Review, Inc.

53

MANUAL FOR THE GRE

2
Line
5

10

15

20

25

30

35

40

54

Though application of evolutionary theory to


the psychology of animals is generally done
without controversy, evolutionary psychology
as it is applied to human psychology is quite
contentious. Proponents of evolutionary psychology believe that psychology must be rooted in
biology. Just as the bodys circulatory, digestive, endocrine, immune, lymphatic, muscular,
nervous, reproductive, respiratory, skeletal, and
urinary systems are evolved adaptations resulting from natural selection or sexual selection, so
too must the seemingly inherent psychological
mechanisms be the result of evolution. These
psychologists believe that natural selection has
engendered many cognitive modules in the brain,
ranging from language-acquisition modules to
cheater-detection modules. Survivability and
sexual selection determine which modules are
passed on.
Some critics offer several objections to extending this application to humans. For instance,
humans evolved during a periodthe Pleistoceneabout which very little essential demographic information on humans is known. Additionally, some accuse evolutionary psychologists
of proffering just-so storiesinternally consistent hypotheses that, nevertheless, have no other
supporting evidence. This, skeptics argue, can
lead to contradictory conclusions. For example,
such behaviors as monogamy can perpetuate
genes, but so too can indelity. Lastly, some critics accuse evolutionary psychology of ethnocentrism since many traits once considered universal
have turned out to be culturally dependent.
Though evolutionary psychology remains controversial, many detractors confess their inexperience and lack of rsthand knowledge with the
discipline. Admittedly, the quality of work in this
eld has been uneven, but, as Edouard Machery
stated, the heuristics and the strategies of conrmation used by evolutionary psychologists are
on a rm grounding.

The Princeton Review, Inc.

Question 1
The primary purpose of the passage is to
explain the origins of evolutionary
psychology
resolve a dispute regarding acceptable
forms of evidence
reconcile the differences between two
methods seeking to explain the same
phenomenon
describe a eld of research and caution
against its dismissal
argue the importance of a debate between
scientists regarding the application of one
methodology to another
Question 2
It can be inferred from the passage that the author
believes which of the following?
Evolutionary psychology has some merit
but has failed to bridge the divide between
the social sciences and the natural
sciences.
Evolutionary psychologists should be
admonished for failing to justify their
hypotheses with more than just-so
stories.
Evolutionary psychology should be
restricted to non-human animals.
Evolutionary psychology should not be
uniformly rejected.
The debate between proponents of
evolutionary psychology and its detractors
is intractable.

LESSON 2 VERBAL

SCRATCH PAPER
Genre (What)

Problem
Question
Conflict
Paradox
Change
Innovation
Discovery

Purpose (Why)

Predict
Recommend
Inform/Explain
Correct
Evaluate

Structure (How)

Cause/Effect
Chronology
Classification
Comparison/Contrast
Steps/Stages

The Princeton Review, Inc.

55

MANUAL FOR THE GRE

2
Line
5

10

15

20

25

30

35

40

56

Some have attributed the dramatic increase


in childhood obesity that has occurred in the
United States from the 1990s throughout
the 2000s to increased consumption of high
fructose corn syrupalso called maize syrup
or glucose-fructose syrupby members of this
demographic. When oligosaccharides, which
are produced by treating cornstarch with alphaamylase, are broken down into simple sugar
glucose by glucoamylase and are then converted
into 42% fructose and 50-52% glucose by mixing
some other sugars along with xylose isomerase,
HFCShigh fructose corn syrupis created.
Increases in consumption of HFCS have been
accompanied by a huge spike in obesity rates
among children and adolescents.
However, studies by the American Medical
Association reveal that absorption by the body
of HFCS is not relevantly dissimilar to the
bodys absorption of sucrose, commonly known
as table sugar. While further epidemiological
studies are needed to test the long-term effects
of such sweeteners, other studies do show that
increased consumption of sugar in general is
a leading cause for the spike in obesity among
children. In the last 20 years, sugar consumption
in the U.S. has increased from 26 pounds per
person per year to 135 pounds per person per
year.
Given the analyses, which suggest that
substantial increases in sucrose consumption
may have negative effects on metabolic
control, it is very likely that these increases are
signicantly responsible for the rise in childhood
obesity. Thus, children should be discouraged
from consuming in large quantities any type of
rened sugar, whether it be table sugar or HFCS.
Accordingly, in an effort to promote public health,
especially among our youth, government policy
makers need not consider removing the corn
subsidies that make HFCS a cheaper alternative
to sucrose for food manufactures but instead
should consider limiting the sale of all foods with
high concentrations of sucrose to children.

The Princeton Review, Inc.

Question 1
The primary purpose of the passage is to
argue against a failed policy
recommend a change in HFCS
consumption
correct a faulty methodology
consider the implications of a study
advocate for the amelioration of childrens
health
Question 2
Which of the following best describes the
organization of the passage?
An explanation is provided, a reason for its
rejection is offered, and further studies are
recommended.
An explanation is put forth; a possible
objection is raised and then dismissed.
An incomplete explanation is revised and a
recommendation based on that revision is
proffered.
A phenomenon is explained; that
explanation is defended and forms the
basis for a policy recommendation.
A point of view is defended against critique
but then dismissed in favor of a more
effective policy.

LESSON 2 VERBAL

SCRATCH PAPER
Genre (What)

Problem
Question
Conflict
Paradox
Change
Innovation
Discovery

Purpose (Why)

Predict
Recommend
Inform/Explain
Correct
Evaluate

Structure (How)

Cause/Effect
Chronology
Classification
Comparison/Contrast
Steps/Stages

The Princeton Review, Inc.

57

Lesson 3 Math

MANUAL FOR THE GRE

PERCENT CHANGE
3

Question 1
A dress that originally sold for $120 now sells for
$96. The new price is what percent less than the
original price ?
8%
12%
20%
24%

TRIGGER

33

Trigger: _______________________________________________________________

Question 2
If x is the percent increase from 3 to 4, for which of the
following does x = y ?
Indicate all such answers.

. y is the percent
.
.

60

The Princeton Review, Inc.

decrease from 4 to 3
y is the percent
decrease from 12 to 8
y is the percent
increase from 6 to 8

LESSON 3 MATH

SCRATCH PAPER
3

RESPONSE

Write the percentage change formula: Percent change = difference 100


original

The Princeton Review, Inc.

61

MANUAL FOR THE GRE

RATIOS
3

Question 3
A can of nuts has almonds and cashews in a ratio of
x : y. If there are z almonds in the can, which of the
following represents the number of cashews?

TRIGGER

y(x + z)

y(z y)

xy

yz

yz

Trigger: _______________________________________________________________

Question 4
A certain jar contains pennies and nickels in a
ratio of 7 : 4, respectively. After three pennies are
removed, the ratio of pennies to nickels is 3 : 2.
Quantity A
12

Quantity B
The number of
nickels in the jar

Question 5
The ratio of a to b is 3 to 4.

62

The Princeton Review, Inc.

Quantity A

Quantity B

a+1

b+1

LESSON 3 MATH

SCRATCH PAPER
3

RESPONSE

Draw a ratio box on your scratch paper.

The Princeton Review, Inc.

63

MANUAL FOR THE GRE

AVERAGES
3

Question 6
The average salary of 12 employees at a certain
rm is $35,000. If the average salary of 8 of the
employees is $40,000, what is the average salary of
the other 4 employees?

TRIGGER

 $25,000
 $27,000
 $27,500
 $28,000
 $30,000

Trigger: _______________________________________________________________

Question 7
The average score for half of the students in a
class on a certain test was 90. The average score
for another fourth of the students was 80. If the
average (arithmetic mean) score for all of the
students was 78, what was the average score for
the remaining students?
52
58
64
70
78

64

The Princeton Review, Inc.

LESSON 3 MATH

SCRATCH PAPER
3

RESPONSE

Draw an Average Pie every time the word average appears in the question.

The Princeton Review, Inc.

65

MANUAL FOR THE GRE

Question 8
Hours Spent Text Messaging

Hours per teenager

Number of teenagers

10

12

14

The table above shows the number of hours


spent text messaging in a week by a group of 13
teenagers. What is the median number of hours of
text messaging per teenager?

TRIGGER

9
10
11
12
13

Trigger: _______________________________________________________________

Question 9
Set A = {5, 5, 4, 4, 1, 6, 3, x, y}
The mode of set A, above, is 5 and the median is 4.
If x > y, and x and y are both integers, what is the
greatest possible value of y ?

66

The Princeton Review, Inc.

LESSON 3 MATH

SCRATCH PAPER
3

RESPONSE

Identify the list and put the numbers in order.

The Princeton Review, Inc.

67

MANUAL FOR THE GRE

RATES
3

Question 10
Rob and David live 200 miles apart. Deciding to
have a picnic, they both start driving at 9:00 a.m.,
traveling in a straight line towards each other. Rob
and David drive at an average speed of 30 and
50 miles per hour, respectively. At what time do they
meet for their picnic?

TRIGGER

11:30 a.m.

1:00 p.m.

1:30 p.m.

Working at a constant rate, Machine A produces


x donuts in 12 hours. Working at a constant rate,
Machine B produces x donuts in 6 hours.
Quantity A
The number of hours it

Quantity B
The number of hours it

will take both machines

will take Machine B to


x
produce
donuts
3

working together to prox


duce
donuts
2

The Princeton Review, Inc.

3:40 p.m.

Trigger: _______________________________________________________________

Question 11

68

2:30 p.m.

LESSON 3 MATH

SCRATCH PAPER
3

RESPONSE

Write down D = R T on your scratch paper.

The Princeton Review, Inc.

69

MANUAL FOR THE GRE

DRILL
3
Question 1 of 8
Q is a set of consecutive odd integers.
Quantity A
The average of set Q






Quantity B
The median of set Q

Quantity A is greater.
Quantity B is greater.
The two quantities are equal.
The relationship cannot be determined from the
information given.

Question 2 of 8
At a certain factory, each worker either drives to
work or takes the bus. The ratio of workers who
take the bus to work to those who drive to work is
2 : 5. If 120 workers drive to work, how many
workers are there at the factory?
300
240
168
48
24

Question 3 of 8
The average age of the members of club K is 22
years. The average age of the members of club Q is
29 years. The average age of the members of both
clubs combined is 27 years.
Quantity A
The number of
members in club K






70

The Princeton Review, Inc.

Quantity B
The number of
members in club Q

Quantity A is greater.
Quantity B is greater.
The two quantities are equal.
The relationship cannot be determined from the
information given.

LESSON 3 MATH

SCRATCH PAPER
3

The Princeton Review, Inc.

71

MANUAL FOR THE GRE

Question 4 of 8

If Dan had increased his average speed by 20 miles


per hour, he would have decreased the time it took
him to drive from his job to a certain restaurant
by 25%. What was Dans actual average speed, in
miles per hour, when he drove from his job to the
restaurant?







30
40
45
50
60

Question 5 of 8

November

October

September

August

July

June

May

Depth of Water (in inches)

Depth of Smith Pond

During a drought, the depth of a pond was


measured each month from May through October.
Each unit on the vertical axis represents 1 inch. If
the depth of the lake decreased 20 percent from
July to August, what was the depth of the lake in
August?

 5 inches
 20 inches
 25 inches
 30 inches
 40 inches

72

The Princeton Review, Inc.

LESSON 3 MATH

SCRATCH PAPER
3

The Princeton Review, Inc.

73

MANUAL FOR THE GRE

Question 6 of 8

Stan drives at an average speed of 60 miles per hour


from Town A to Town B, a distance of 150 miles.
Ollie drives at an average speed of 50 miles per hour
from Town C to Town B, a distance of 120 miles.
Quantity A
Amount of time Stan
spends driving

Quantity B
Amount of time Ollie
spends driving

Question 7 of 8
The average of 2 numbers is x. If the average of x
and y is z, which of the following is y in terms of x
and z ?
x+ z
2
x+ z
3
2z x
z
3

x
2

3z 2x

Question 8 of 8
Earthquake Fault
Zones

Average Annual
Frequency of Earthquakes
1986-1993

Zone One
Zone Two
Zone Three
Zone Four
Zone Five

x
8.7
5.3
5.7
y

In the chart above, if the mean frequency of


earthquakes in Zones One, Two, and Three is 8.0,
and the mean frequency of earthquakes in Zones
Four and Five is 5.5, then how much less than the
mean of the ve annual earthquake frequencies is
the mode of the ve annual earthquake
frequencies?
1.7

74

The Princeton Review, Inc.

5.3

5.5

7.0

8.0

LESSON 3 MATH

SCRATCH PAPER
3

The Princeton Review, Inc.

75

MANUAL FOR THE GRE

SUMMARY
TRIGGER
TRIGGER
TRIGGER
TRIGGER
The Princeton Review, Inc.

median

speed
rate

difference
100
original

Draw ratio box on scratch


paper.

Draw an Average Pie every


time the word average appears
in the question.

Identify the list and put the


numbers in order.

Write formula: D = R T

RESPONSE

average

Percent change =

RESPONSE

ratio

TRIGGER

percent increase/more

Write formula:

RESPONSE

percent decrease/less

RESPONSE

76

percent change

RESPONSE

Lesson 3
Verbal

MANUAL FOR THE GRE

Question 1

Bob is well known for being _________ because he


frequently gives money to charities.

.
.
.
.
.

SCRATCH PAPER
Your word/phrase:

critical
stingy
cunning
munificent
famous

Ignore the answer choices. Dont plug the words into


the sentence.

Steps for Sentence Equivalence Questions


1.
2.
3.

78

Find the Story.


Come up with your own word or phrase for the blank. Write that word or phrase down on your
scratch paper.
Check each answer choice and use your scratch paper:
an answer that sort of matches your word
an answer that does not at all match your word
? any word you dont know
for a maybe

The Princeton Review, Inc.

LESSON 3 VERBAL

GET A CLUE
Who or what is the blank talking about?
What information does the sentence give you about that person or thing?

The clue is the word or phrase in the sentence that indicates


what word or idea must go in the blank.

His greatest talent was his _________ : his ability to lie to anyone.
Suddenly he _________ , and no one could tell where he had disappeared to.
Acclaimed by several important artists as a prodigy, Van Vliet was a sculpting _________ .
The _________ professor was so talkative that his rambling lectures would continue long after the students
had left the lecture hall.
Tamson was so gifted a singer that her colleagues were often dazzled by her _________ and failed to
appreciate her other talents.
Art critics have characterized Jacksons latest work as a _________ of different ideas, all thrown together
with little thought of any unifying theme.
The _________ stories in Browns novels, which were written in the early years after the founding of the
United States of America and are frequently the subject of contextual analysis by historians, are noted for
their dark, forbidding tone.

The Princeton Review, Inc.

79

MANUAL FOR THE GRE

Question 2

Filmmaking is a _________ effort, since the director,


cinematographer, writer, editor, and many others
must work together to produce a movie.

.
.
.
.
.
.

creative
lucrative
collaborative
joint
concentrated
glamorous

Question 3
As CEO, Mr. Bluvband distinguished himself
primarily because he was more _________ than his
peers; he remained calm and resolute even during
times of immense volatility in the market at large.

.
.
.
.
.
.

80

prosperous
stalwart
dismayed
prescient
steadfast
nave

The Princeton Review, Inc.

SCRATCH PAPER
Your word/phrase:

LESSON 3 VERBAL

Question 4

SCRATCH PAPER

Although most species of sh off the southwest


coast of Africa have died off due to the oxygen-poor
and jellysh-rich waters, the bearded goby is
actually _________ .

.
.
.
.
.
.

Your word/phrase:

thriving
destructive
withering
transitory
ourishing
picturesque

I really like you, and ______________________________________________.


I really like you, but ______________________________________________.

Contrasts

Agrees

although
though
despite
however
nonetheless
nevertheless
instead
rather
whereas
while
but
yet

thus
therefore
not only . . . but also
insofar
because
and
;
:
since
consequently

She used to be poor; now shes ______________________________________.


His early work was boring, so its refreshing to see that his latest piece is ______
_______________________________________________________________.

The Princeton Review, Inc.

81

MANUAL FOR THE GRE

Question 5

Some pundits postulate that the end of the


Cold War _________ one of the major political
parties; many voters had previously viewed that
party as vital because of its role in the ght against
Communism.

.
.
.
.
.
.

fragmented
bolstered
encouraged
engendered
vitiated
attenuated

Question 6
As soon as Brett stepped off stage, he slipped out of
his brash and outspoken stage persona and returned
to his usual _________ self.

.
.
.
.
.
.

82

timorous
heady
difdent
orid
fastidious
blunt

The Princeton Review, Inc.

SCRATCH PAPER
Your word/phrase:

LESSON 3 VERBAL

DRILL
Question 1 of 5
The stunning array of colors present in our natural
world would suggest a seemingly _________ palette
for paint manufacturers to draw on for inspiration,
but only a small fraction of these colors are actually
available.

.
.
.
.
.
.

SCRATCH PAPER

Your word/phrase:

xed
innite
innumerable
realistic
limited
essential

Question 2 of 5
Jamess _________ work associates were among the
many who trusted his nancial savvy and testied
on his behalf in the hearing, only to nd themselves
stunned and devastated by his confession on the
nal day of the proceedings.

.
.
.
.
.
.

cynical
unwavering
petty
recent
steadfast
capricious

The Princeton Review, Inc.

83

MANUAL FOR THE GRE

Question 3 of 5

Colberts pompous character is crafted with


such _________ that some critics mistake his satire
for mere bombast.

.
.
.
.
.
.

insouciance
guile
originality
cunning
literacy
refulgence

Question 4 of 5
Unfortunately, Jacksons public disavowal of the
group was _________ by his ongoing support of the
groups leaders.

.
.
.
.
.
.

undermined
fortied
reviewed
braced
subverted
retained

Question 5 of 5
The studio head, naturally _________ , lets his
underlings speak for him during most meetings,
remaining silent until absolutely necessary.

.
.
.
.
.
.

84

taciturn
laconic
solicitous
impertinent
loquacious
blithe

The Princeton Review, Inc.

SCRATCH PAPER
Your word/phrase:

LESSON 3 VERBAL

STEPS FOR SENTENCE EQUIVALENCE


QUESTIONS
1.

Find the Story.

2.

Come up with your own word or phrase for the


blank. Write it down on your scratch paper.

3.

Check each answer choice:


an answer that sort of matches your word
an answer that does not at all match your word
? any word you dont know
for a maybe

The Princeton Review, Inc.

85

MANUAL FOR THE GRE

3
5

10

15

20

25

30

35

40

45

86

Because democracies allow their citizens to


elect leaders and sometimesthrough referendumvote on particular laws, it may seem that
the primary result of political democratization is
the increase in freedom for the countrys citizens.
But in spite of the fact that democracies ostensibly allow citizens to place limits on the power
exercised by their leaders, the 20th century saw
the rise of what Fareed Zakaria termed illiberal
democraciesgovernments elected through
democratic elections but nonetheless tyrannicalwhich deprive their citizens of rights and
freedoms.
This phenomenon can be seen in various
regions of the globe ranging from Peru to Sierra Leone to Slovakia to Pakistan. A particularly
infamous example is Adolf Hitler, who became
chancellor of Germany through free election. One
unexpected result is that tyrannical regimes can
be far more tolerant than their liberal counterparts.
This incongruity is not mere irony or accident,
but rather the result of the absence of one essential component for a free society. Giovanni
Sartori notes how most Third World countries
thatwhile still impoverishedproclaimed themselves democracies became dictatorships within
a decade. In contrast, autocratic regimes that cultivated economic freedom, even if only for elites,
inevitably became democracies by the mid-20th
century. Consider the democratic evolution of
Great Britain, Denmark, Belgium, and the Netherlands, in contrast with the democratic evolution
of France, Austria, and Germany. The evolutions
of the former were far more peaceful compared
to those of the latter. History has shown that in
even the most undemocratic regimes, once the
seeds of an economically independent middle
class have been planted, the change to a free,
democratic society is inevitable. The bestconsolidated democracies in Latin America and
East Asia were all ruled by military juntas. But
by opening their economieshowever slowly
the process made the government more liberal
and the economically prosperous citizenry more
demanding of limits of power; enfranchisement
became inevitable. It is no coincidence, then, that
an economically independent middle class is glaringly absent in failed democracies that degenerated into dictatorships.

The Princeton Review, Inc.

Question 1
All of the following are stated by the author
as possible characteristics of countries with
democratically elected regimes EXCEPT
enfranchised populaces

 limits on powers of leaders


 despotism
 freedom of religion
 economic independence for its middle
class
Question 2
Consider each of the choices separately and select
all that apply.
The author suggests which of the following about
democracies?

. They may be more resilient when


buttressed by a strong middle class.

. They are not sufcient for securing certain


.

freedoms for their citizens.


During the 20th century, they usually
devolved into tyrannical regimes.

LESSON 3 VERBAL

SCRATCH PAPER
Answering the Questions

1. Translate the question.


2. Find proof in the passage.
3. Answer the question in your own words.
4. POE. Look to ELIMINATE.

Wrong answers
contradict the passage
dont answer the question
are NOT found in the text

The Princeton Review, Inc.

87

MANUAL FOR THE GRE

Question 3

Question 4

In the context in which it appears, incongruity


(line 21) refers to

The primary purpose of the passage is to

 the fact that some democracies only confer


rights to the elite and not to the masses
 the fact that democracies inevitably
devolve into despotic regimes
 the fact that some despotic regimes have
certain liberal characteristics that some
democracies lack
 the fact that the middle class is largely
absent from failed democracies
 the fact that some countries have yet to
evolve into democratic regimes

88

The Princeton Review, Inc.

 compare two outcomes that result from


implementing a particular system of
government
 contrast two forms of government
 explain why one form of government is
superior to another
 discuss why democracies are not always
capitalistic
 explain why an apparent contradiction
occurs

LESSON 3 VERBAL

SCRATCH PAPER
Genre (What)

Problem
Question
Conflict
Paradox
Change
Innovation
Discovery

Purpose (Why)

Predict
Recommend
Inform/Explain
Correct
Evaluate

Structure (How)

Cause/Effect
Chronology
Classification
Comparison/Contrast
Steps/Stages

The Princeton Review, Inc.

89

MANUAL FOR THE GRE

3
5

10

15

20

Theorists are divided about the cause of the


Permian mass extinction. Some hypothesize that
the impact of a massive asteroid caused the
sudden eradication of most species. However, a
look at the carbon-isotopic record suggests that
existing plant communities were decimated and
revived several times. To produce such a pattern
would require a succession of asteroid strikes
thousands of years apart. Other theorists have
proposed that volcanic explosions raised the
CO2 levels, leading to intense global warming.
One problem with this theory is that it cannot
explain the mass marine extinctions at the end
of the Permian period. A new theory proposes
that the combination of rising concentrations of
toxic hydrogen sulde in the worlds oceans and
gradual oxygen depletions in the surface waters
caused the extinctions. Fortunately, this theory
is testable. If this theory is true, then oceanic
sediments from the Permian period will yield
chemical evidence of a rise in hydrogen sulde
consuming bacteria.
Question 1
The primary purpose of the passage is to

 consider several hypotheses concerning


the cause of the Permian mass extinctions

 discuss the strengths and weaknesses of


the asteroid hypothesis concerning the
cause of the Permian mass extinctions
 propose that theories regarding the cause
of the Permian mass extinctions be tested
 argue the Permian mass extinctions
could not have been caused by a volcanic
explosion
 describe one reason that a rise in hydrogen
sulde would cause massive marine
extinctions
Question 2
Select the sentence that best illustrates the reason
one theory may be superior to the other theories
mentioned in the passage.

90

The Princeton Review, Inc.

Question 3
According to the passage, the author would most
likely agree with which of the following?

 The cause of the Permian mass extinction


is not controversial.
 The Earth experienced a series of
successive meteor strikes that destroyed
plant life.
 Marine life must have survived the Permian
mass extinction.
 The Permian mass extinction was caused
by depleted oxygen that accompanied
increases in toxic hydrogen sulde in the
oceans.
 There is no conclusive explanation for the
Permian mass extinction.

LESSON 3 VERBAL

SCRATCH PAPER
3

The Princeton Review, Inc.

91

MANUAL FOR THE GRE

3
5

10

15

20

25

30

35

40

It is frequently assumed that freeing schools


from the rigid rules, regulations, and statutes
that govern them will have a revolutionary effect
on academic achievement. For instance, it has
been suggested that such schools could develop
more effective teaching methods that could then
be replicated in other schools. Charter schools,
public schools that operate under a contract (or
charter), were given just such an opportunity. In
1991, when Minnesota passed the rst charter
school law, many critics warned of the deleterious effects that such free-wheeling schools could
have on the academic achievement of students.
Thus, while public opinion differed concerning the social desirability of charter schools, all
agreed that there would likely be a pronounced
effect.
Educators, particularly those interested in
educational reform, now seriously question the
degree to which charter schools have made an
impact. They conclude that freedom from many
of the policies and regulations affecting traditional public schools and the concomitant control
over decisions that guide the day-to-day affairs of
the school have not resulted in equally dramatic
changes in students academic abilities. In many
states, charter schools are less likely to meet
state performance standards than traditional
public schools. It is, however, impossible to know
whether this is because of the performance of
the schools, the prior achievement of the students, or some other factor.
Metrics for educational accountability have
changed considerably in the past decade; performance on state-mandated tests is increasingly used as a measure of student performance.
Fundamentally, the challenging conditions under
which schools operate, be they traditional or
charter schools, have changed little: the struggle
for resources, low pay for teachers, answerability to multiple stakeholders, and the difculty of
meeting the educational requirements of children
with special needs all persist.

Question 1
Which of the following statements best summarizes
the main point of the passage?

 Assessments of charter schools


performance have reinforced the position
that rigid rules and regulations are stiing
academic achievement.
 Freeing schools from some regulations has
caused a positive change in education.
 Charter schools have engendered a whole
new approach to educating children.
 Freeing schools from rules and regulations
has not had the deleterious effect that
some critics had feared.
 Charter schools, despite their merits, fail
to overcome the long-standing problems in
public education.
Question 2
Which of the following best describes the function
of the concluding paragraph of the passage?

 It sums up the general points concerning


charter schools made in the entire passage.
 It draws a conclusion concerning the
challenges that schools face that goes
beyond the information in the previous
paragraphs.
 It restates the point concerning the
successes that schools have achieved.
 It qualies the authors agreement with
those academics who question the degree
to which charter schools have made an
impact.
 It qualies the authors rejection of those
academics who question the degree to
which charter schools have made an
impact.
Question 3
In the rst paragraph, select the sentence that best
illustrates an example of a hypothetical benet.

92

The Princeton Review, Inc.

LESSON 3 VERBAL

SCRATCH PAPER
3

The Princeton Review, Inc.

93

MANUAL FOR THE GRE

SUMMARY
Answering the Questions
1.

Translate the question.

2.

Find proof in the passage.

3.

Answer the question in your own words.

4.

POE. Look to ELIMINATE.

Wrong answers

94

The Princeton Review, Inc.

contradict the passage

dont answer the question

are NOT found in the text

Lesson 4
Math

MANUAL FOR THE GRE

SCRATCH PAPER
GEOMETRY BASIC STEPS
4

96

1.
2.
3.

Draw It.
Label Everything!
Write out Formulas.

The Princeton Review, Inc.

LESSON 4 MATH

SCRATCH PAPER

The Princeton Review, Inc.

97

MANUAL FOR THE GRE

TRIANGLES
Question 1
B

D
ABCD is a square.

TRIGGER

Quantity A
The length of
diagonal AC

Quantity B
12

Trigger: _______________________________________________________________

Question 2
Y

Triangle XYZ is an equilateral triangle.


Quantity A
The area of XYZ

98

The Princeton Review, Inc.

Quantity B
8

LESSON 4 MATH

SCRATCH PAPER

Identify your triangles and label the sides!

a 2

30
2a
a 3

45
a

RESPONSE

45

60

The Princeton Review, Inc.

99

MANUAL FOR THE GRE

Question 3
B

30

60
A

What is the perimeter of triangle ABC in terms of a ?


2a + 3

a(3 + 3 )

3a + a 2

2a 3

Question 4
B

Triangle ABC is an isosceles triangle and AB = 6


Quantity A
The perimeter
of triangle ABC

Quantity B
The area of
triangle ABC

Question 5
B
105

30

In the triangle above, if BC = 4 2 , then AB =


6
4 3
8
4 6
10

100

The Princeton Review, Inc.

2 a+a

LESSON 4 MATH

SCRATCH PAPER

The Princeton Review, Inc.

101

MANUAL FOR THE GRE

CIRCLES
Question 1
A

B
C

O is the center of the circle shown above. If the


measure of angle AOC is 60, then the length of arc
ABC is what fraction of the circumference?

TRIGGER

Enter your answer as a fraction.

Trigger: _______________________________________________________________

Question 2
B
4
O

225

O is the center of the circle shown above. What is


the area of the shaded region?
3

102

The Princeton Review, Inc.

16

LESSON 4 MATH

SCRATCH PAPER

RESPONSE

Central Angle
sector
arc
=
=
360
circumference area

The Princeton Review, Inc.

103

MANUAL FOR THE GRE

Question 3
B
A
C

9
O

O is the center of the circle above. If arc ABC has a


length of 5, what is the measure of angle AOC ?
60
90
100
110
120

Question 4
B
A
C

Note: Figure not drawn to scale.


In the circle above, chord AC has a length of 6. If
the length of arc ABC is equal to one-sixth of the
circumference of the circle, what is the area of the
circle?
6
12
24
36
48

104

The Princeton Review, Inc.

LESSON 4 MATH

SCRATCH PAPER

The Princeton Review, Inc.

105

MANUAL FOR THE GRE

SHADED REGIONS AND STRANGE-LOOKING SHAPES


Question 1
C

4
B

In the gure above, AC = CE = AE = 8. If B and D


are the midpoints of AC and CE, respectively, what
is the area of the shaded region?

TRIGGER

12 3

16 3

24 3

64

Trigger: _______________________________________________________________

Question 2
2m

In the gure above, if each short segment has


length 2m, and all segments intersect at right
angles, then what is the area of the gure?
6m
106

The Princeton Review, Inc.

6m2

15m2

18m2

60m2

LESSON 4 MATH

SCRATCH PAPER

Shaded = Total Unshaded


Draw lines (split, connect, draw radii & drop heights, etc.).
If all else fails, play with the gures!

RESPONSE

Find familiar shapes.

The Princeton Review, Inc.

107

MANUAL FOR THE GRE

Question 3
G

10
F

10

What is the area of quadrilateral EFGH shown above?


24 + 25 3
24 + 50 3
48 + 50 3
72
It cannot be determined from the
information given.

Question 4
W

In the gure above, side XY of square WXYZ is equal


to 2. If Z is the center of the circle, then what is the
area of the shaded region?
4
8 2
16 4
4 + 2
8+

108

The Princeton Review, Inc.

LESSON 4 MATH

SCRATCH PAPER

The Princeton Review, Inc.

109

MANUAL FOR THE GRE

OVERLAPPING SHAPES
Question 1

4
B

Note: Figure not drawn to scale


Rectangle ABCD has length 6 and width 8. What is
the area of the circle?

TRIGGER

10
20
25
64
100

110

The Princeton Review, Inc.

Trigger: _______________________________________________________________

LESSON 4 MATH

SCRATCH PAPER

RESPONSE

Figure out what the two shapes have in common.

The Princeton Review, Inc.

111

MANUAL FOR THE GRE

Question 2
B
x

D
In the gure above, if A and C are the centers of the
two circles, then x =
30
40
45
60
90

Question 3
B

D
AD || BC

Quantity A
The area of
triangle ABC






112

The Princeton Review, Inc.

Quantity B
The area of
triangle BCD

Quantity A is greater.
Quantity B is greater.
The two quantities are equal.
The relationship cannot be determined from the
information given.

LESSON 4 MATH

SCRATCH PAPER

The Princeton Review, Inc.

113

MANUAL FOR THE GRE

DRAW & REDRAW


Question 1

A triangle has sides of lengths 3 and 4.

TRIGGER

Quantity A
The length of
the third side

Quantity B
5

Trigger: _______________________________________________________________

Question 2
c
d
a

g
h

Note: Not drawn to scale.


In the gure above, if a = e, then which of the
following must be true?
Indicate all such statements.

.
.
.

114

The Princeton Review, Inc.

a=f
b=f
c=h

LESSON 4 MATH

SCRATCH PAPER

RESPONSE

Draw it twice. Exaggerate the differences.

The Princeton Review, Inc.

115

MANUAL FOR THE GRE

STRETCH & SQUASH


Question 1

6.1

6.1

6.1

6.0

6.1

6.1

Quantity A
f

Quantity B
g

When redrawing your shape, stretch and squash.


Exaggerate the differences.

Question 2
Two quilts are equal in area. The rst quilt is made
up of 25 patches, each of which has an area of x
square inches. The second quilt is made up of 14
patches, each of which has an area of y square
inches.
Quantity A
x

Quantity B
y

Question 3

(x, y)
a

x<y
Quantity A
a
116

The Princeton Review, Inc.

Quantity B
45

LESSON 4 MATH

SCRATCH PAPER

The Princeton Review, Inc.

117

MANUAL FOR THE GRE

GEOMETRY TIMED DRILL


Question 1 of 10

M
x

l1

40
65

l2

In the gure above, l1 || l2. What is the value of x ?


60

75

105

115

140

Question 2 of 10
E

B
120
C
A

In the gure above, C is the midpoint of both


segment AE and segment BD. If AE = BD, and
BCE = 120, then x =
30

45

50

60

75

Question 3 of 10
q
p

r
n

In the gure above, four lines intersect as shown. If


n = 60, what is the value of p + q + r ?
180

118

The Princeton Review, Inc.

240

300

420

It cannot be
determined from the
information given.

LESSON 4 MATH

SCRATCH PAPER

The Princeton Review, Inc.

119

MANUAL FOR THE GRE

a+

b+

Question 4 of 10

Quantity A
a

Quantity B
b

Question 5 of 10
P

In rectangle PQRS above,

PQ

. The sum of
QR
3
PQ + QR + RS is what percent of the perimeter of
the rectangle?
55

9
2
66 %
3
70%
75%
5
85 %
7
Question 6 of 10
Quantity A
The perimeter of a
rectangle with a side
of length 15

120

The Princeton Review, Inc.

Quantity B
The perimeter of a
square with a side of
length 8

LESSON 4 MATH

SCRATCH PAPER

The Princeton Review, Inc.

121

MANUAL FOR THE GRE

Question 7 of 10

The two circles with centers at Q and S both


have radius r.
Quantity A
One-half the area of
the circle with center Q






Quantity B
The shaded region

Quantity A is greater.
Quantity B is greater.
The two quantities are equal.
The relationship cannot be determined from the
information given.

Question 8 of 10

Note: Figure not drawn to scale.


In the gure above, the area of the larger circular
region is twice the area of the smaller circular
region. If the diameter of the larger circular region is
x, then, in terms of x, what is the area of the smaller
circular region?

x2
2
x2
4
x2
8
x
2
x
4

122

The Princeton Review, Inc.

LESSON 4 MATH

SCRATCH PAPER

The Princeton Review, Inc.

123

MANUAL FOR THE GRE

Question 9 of 10
G
H

K
I

In the rectangular solid depicted above, HI = 5,


IJ = 12, and JK = 3.
Quantity A
The area of rectangle HGKJ






Quantity B
39

Quantity A is greater.
Quantity B is greater.
The two quantities are equal.
The relationship cannot be determined from the
information given.

Question 10 of 10
Quantity A
The volume of a cylinder
that has a circular base
with a radius of 3 inches
and a height of 5 inches






124

The Princeton Review, Inc.

Quantity B
The volume of a cylinder
that has a circular base
with a radius of 4 inches
and a height of 3 inches

Quantity A is greater.
Quantity B is greater.
The two quantities are equal.
The relationship cannot be determined from the
information given.

LESSON 4 MATH

SCRATCH PAPER

The Princeton Review, Inc.

125

MANUAL FOR THE GRE

TRIGGER
TRIGGER

Arc, sector, or a fraction of


a circle

Find the 45-45-90 triangle


(you know its in there somewhere).

Find the 30-60-90 triangle


(you know its in there somewhere).

Write your formula:

RESPONSE

Right triangle with 30, 60, 3 ,


a big side that is twice the small
side, or the height of an equilateral
triangle

RESPONSE

Right triangle with 45 or


or diagonal of a square

RESPONSE

TRIGGER

SUMMARY

angle
F

360

arc
area sector .
=
circumference area circle

TRIGGER
TRIGGER

Overlapping shapes

Find the elements the two shapes


have in common.

TRIGGER

Geometry on Quant Comp or


Must Be questions

Draw the shape twice.


Exaggerate the differences.

RESPONSE

Shaded regions or strange-looking


shapes

RESPONSE

126

RESPONSE

Find familiar shapes.

The Princeton Review, Inc.

Shaded = Total Unshaded.


Draw lines (split, connect,
draw radii, drop heights, or
otherwise play with the shape).

Lesson 4
Verbal

MANUAL FOR THE GRE

ATTACKING THE QUESTIONS


4

1.
2.
3.
4.

Translate the Question


Proof: Find proof in the passage
Predict: Answer the question in your own words
POE: Look to eliminate

How GRE Constructs a Typical Reading Comp Question


1.

Question about a specic part of the passage


or the passage overall. What the passage
states, WHY the passage states something,
HOW the author feels about something, or
WHAT would be similar to the passage, etc.
Words from the wrong part of the
passage (doesnt answer the question)
Words from the passage made too
extreme (not in the text)
Words combined with something NOT
from the passage (not in the text)
Correct information paraphrased and
disguised
Patchwork of words and phrases taken
directly from the passage

How to Avoid Falling Into Traps

Predict what the answer will do before looking at the answer choices.
Read all five answers in full. Use a two-pass approach through the answer choices.
Look to eliminate, not to justify.
Remember: Half Bad = All Bad.

Common Wrong Answer Types


1.

2.

3.

128

Doesnt answer the question


Too broad or too narrow
Wrong location (peripheral or wrong paragraph)
Contradicts the passage
Contradicts a fact in the passage
Goes against the main point or purpose
Contradicts the structure/chronology in the passage
Not in the text
Might be true but not supported
Contains information from outside the passage (predictions or judgments)
Extreme languageabsolutes, superlatives, and extreme words (must, always, all) that go
beyond the passage

The Princeton Review, Inc.

LESSON 4 VERBAL

Down to 2?
1)
2)

Compare the remaining answers to each other. Whats the same? Whats different?
Go back to the passage. Which answer choice most agrees with the main idea?

Still struggling? Just mark it, move on, and come back to it after two questions.

If you cant put your nger on a single word, phrase, or


sentence that proves your answer choice, you cannot pick it.
Its wrong.

The Princeton Review, Inc.

129

MANUAL FOR THE GRE

Line

10

15

20

25

30

35

130

The Princeton Review, Inc.

One of several frames of reference utilized by


living creatures in the effort at orientation is the
direction of gravity. In human beings, this orientation is provided by the vestibular system of the
inner ear, a labyrinth of chambers comprising
the semicircular canals, the utricle, and the saccule. The three semicircular canals are oriented
perpendicularly to one another and provide the
locus of balance. In essence, these canals form a
system of Cartesian coordinate axes along which
the brain maps the bodys horizontal, vertical,
and lateral displacements. A mass of particulate
material presses down on membranes, further
allowing the mind to register linear motion and
the direction of gravity. By utilizing this system of
mental signals, we manage to maintain a sound
sense of orientation despite the fact that our
heads are seldom perfectly perpendicular to the
plane of gravitational pull.
Yet this complex system has its imperfections.
As anyone will attest, watching television while
lying sideways is taxing on the brain, while reading at the same inclination is all but impossible
(without tilting the book to match the angle of
sight). Another instance of the systems failure is
motion sickness.
When signals from the retinas frame and
signals from the inner ears frame fail to coincide, the result is motion sickness. The brain is
ooded with contradictory signals: while the eye
indicates that the body is at rest, the inner ear
signals just the opposite. The simple cure for
this is to restore both systems of signals to synchrony. This is accomplished by redirecting the
gaze to the exterior of the moving vehicle, allowing the eye to match the information contributed
by the vestibular system.

LESSON 4 VERBAL

SCRATCH PAPER

The Princeton Review, Inc.

131

MANUAL FOR THE GRE

Question 1

Question 4

According to the passage, motion sickness is


induced by

The main function of the passage as a whole is to

delayed compensation of the vestibular


system
asynchronous pairing of information from
two systems
the non-perpendicular orientation of the
head with respect to the body
a failure to register linear motion and the
direction of gravity
disorientation caused by pressure in the
inner ear
Question 2

Question 5

Consider each of the choices separately and select


all that apply.

Which of the following best describes the


organization of the passage?

According to the passage, the vestibular system


of the inner ear allows humans to accomplish the
following:

. Track forward movement


. Pinpoint the origin of a sound
. Maintain spatial orientation
Question 3
It can be inferred from the passage that the utricle
and the saccule
are solely responsible for the maintenance
of balance
correct for gravitational orientation
provide axes of spatial orientation
are integral with the semicircular
canals
function only in tandem with visual signals

132

account for the failure of the vestibular


system of the inner ear
reconcile discordant theories about spatial
orientation
investigate the impact of stresses on the
vestibular system of the inner ear
consider the limitations of one bodily
system
describe the various frames of reference
used to orient the body

The Princeton Review, Inc.

A system is described, details are provided,


and possible alternative explanations are
considered.
A system is described, and its
imperfections are explained and contrasted
with another systems.
A system is described, its components
are enumerated, and an explanation for its
purpose is introduced.
A system is described, and examples
suggesting that the system is inadequate
are considered.
A system is described and weaknesses of
that system are then exposed.

LESSON 4 VERBAL

SCRATCH PAPER

The Princeton Review, Inc.

133

MANUAL FOR THE GRE

Line

10

15

20

25

30

35

134

The Princeton Review, Inc.

The wealth of morphological, phonetic, and


word similarities among certain languages has
led linguists to recognize the unity of the welldened family of languages called the Aryan or
Indo-European family. Yet even this latter term
is largely a misnomer. This group of languages
spreads over an enormous range virtually without
interruption, reaching from Central Asia to the
fringes of westernmost Europe. The westernmost terminus of the family is Celtic, while its
easternmost representatives were the Tokharian
languages, a pair of tongues once spoken by the
residents of the Tarim River Basin in Western
China and unearthed in documents written more
than a thousand years ago.
So remarkable and denite are the similarities
among these languages that linguists are convinced they all derived from an earlier language
spoken by some community in the prehistoric
past. While we know that Latin began as a rustic
dialect in the province of Latium, no one knows
where proto-Aryan was rst spoken. Some speculate that it was rst used in Southern Russia,
while still others point to the Iranian plateau as a
potential cradle. Though some philologists
believe that the Old Indic and Persian of the
Avesta contain the most archaic features of
Aryan found to date, this does not necessarily x
the habitat of these early Aryan-speaking peoples
closer to Asia than to Europe. Consider Icelandic.
Though this language has strayed far from its
birthplace, it preserves many of the characteristics discarded by those who remained behind.
From the existing evidence, only one thing
seems certain. By the time of the Vedic hymns,
the rst recorded instance of Aryan, those tribes
speaking this early language had already begun
their widespread dispersal.

LESSON 4 VERBAL

SCRATCH PAPER

The Princeton Review, Inc.

135

MANUAL FOR THE GRE

Question 1

Question 3

The main purpose of this passage is to

The author mentions the Tokharian languages


primarily in order to

account for a difference


illustrate a difculty
advocate a change
refute a hypothesis
dene similarities

date them to a far earlier period than Celtic


suggest that they possess many of the
same features as did the Old Indic and
Persian of the Avesta
prove that they were once spoken over a
far greater range than they are today
undermine the accuracy of the name
Indo-European for their particular family of
languages
explain the remarkable similarities among
the languages spoken by residents of the
Tarim River Basin

Question 2
Which of the following situations is most nearly
analogous to the history and unique characteristics
of Icelandic as described in the passage?
A nomadic tribe which resides primarily
in one part of the Sahara is known to sing
songs from a distant part of the Sahara.
Two communities of people develop similar
styles of pictographic writing despite never
having had contact with one another.
Descendants of the French explorers who
rst settled an Atlantic island routinely use
words, phrases, and forms of address longabandoned in contemporary France.
One group of South American Indians
speaks a language that is a unique
assimilation of several distinct modern
languages.
Modern inhabitants of a city speak a
dialect which retains many of the words
abandoned by speakers of a different
dialect in a distant city.

136

The Princeton Review, Inc.

Question 4
Consider each of the choices separately and select
all that apply.
According to the passage, Indo-European languages
are characterized by

. similarities in the structures of words and


.
.

in the sounds employed


widespread recorded instances in
prehistoric literature
preservation of characteristics discarded by
earlier speakers

LESSON 4 VERBAL

SCRATCH PAPER

The Princeton Review, Inc.

137

MANUAL FOR THE GRE

SUMMARY
1)

Translate the Question / Make it a question: What


info do they want?

2)

Proof: Where is this info? Map or Passage? Which


paragraph?

3)

Predict: What should the correct answer do or say?

4)

POE: Look to eliminate, not to justify.

Wrong answers:

Contradict the passage

138

The Princeton Review, Inc.

Dont answer the question

Are not found in the text

Lesson 5
Math

MANUAL FOR THE GRE

THE CHART
TOTAL EXPENDITURES AND SUMMARY OF
COMMUNITY SERVICE EXPENDITURES FOR CITY T
Total Expenditures in City T, 1998

Community Services
44%

Public Transit
14%

Urban Planning
and Development
1%
Corporate Services
4%

Other
12%

Culture and Tourism


6%
Police
8%

Public Works
11%

100% = $5.6 billion


Summary of Community Services Expenditures by Category in City T

Childrens Services
Retirement Homes
Hostels
Housing
Libraries
Public Health
Social Development
Social Services
Total

140

The Princeton Review, Inc.

1997
$211,000,000
$181,000,000
$48,000,000
$264,000,000
$121,000,000
$89,000,000
$9,000,000
$906,000,000

1998
$228,000,000
$176,000,000
$62,000,000
$624,000,000
$117,000,000
$96,000,000
$8,000,000
$1,153,000,000

$1,829,000,000

$2,458,000,000

LESSON 5 MATH

GET YOUR BEARINGS


Before you start the questions, spend a few seconds looking the charts.

Scroll down to find all the charts.


Read titles and legends.
Check units.

WORKING THE QUESTIONS


1.
2.
3.
4.

Read the question carefully. Which chart is the question about?


Go back to the chart to locate information specific to the question. Use your scratch paper as a
ruler if necessary.
Stay organized. Label all numbers and the results of EVERY step.
Estimate before you calculate. Use rounding, estimation, and common percent-to-fraction
conversions to arrive at a rough answer.

The Princeton Review, Inc.

141

MANUAL FOR THE GRE

Question 1
Which one of the following Community Service
programs had the greatest percent increase in
expenditure from the 1997 budget to the 1998
budget in City T ?
Childrens Services
Hostels
Public Health
Social Development
Social Services

Question 2
The amount by which City Ts Community Service
expenditures in 1998 exceeded Public Works
expenditures in 1998 was what percent of the
expenditures for Public Works in 1998 ?
Disregard the percent sign when entering your
answer.

Question 3
Which of the following can be inferred from the graph
or table?
Indicate all such statements.

.
.
.

142

The Princeton Review, Inc.

In 1998, more was spent on Social Services than


on any other area.
In 1998, Community Service expenditures in City
T exceeded expenditures for the next four most
highly funded areas combined.
In both 1987 and 1998, Libraries received greater
funding than did Public Health.

LESSON 5 MATH

SCRATCH PAPER

The Princeton Review, Inc.

143

MANUAL FOR THE GRE

UNITED STATES UNEMPLOYMENT RATES AS A PERCENT OF


THE POPULATION 16 YEARS AND OLDER BY REGION
1998
1999

5.7

5
Percent

5.4
5.2
4.9

5
4.7
4.4

4.4

4.1

3.7

Northeast

3.6

Midwest

South

West

Pacific

Region
Question 4
Which region had the smallest decrease in its
unemployment rate?
Northeast

Midwest

South

Question 5
In 1999, if a total of 1.2 million people were
unemployed in the Northeast region, what was the
overall population of people 16 years or older in the
United States that year, in millions?
million

Question 6
Which of the following regions had the greatest
percent decrease in unemployment rate from 1998
to 1999 ?

 Northeast
 Midwest
 South
 West
Pacic

144

The Princeton Review, Inc.

West

Pacic

LESSON 5 MATH

SCRATCH PAPER

The Princeton Review, Inc.

145

MANUAL FOR THE GRE

TRIGGER

STANDARD DEVIATION

Trigger: _______________________________________________________________

Question 1
Which of the following lists of data has the greatest
standard deviation?
10,10,10

9, 10, 11

5, 10, 15

Question 2
Set A is a set of 10 integers, each of which is
between 0 and 100.
Set B is a set of 10 integers, each of which is
between 100 and 200.
Quantity A
The standard
deviation of Set A

Quantity B
The standard
deviation of Set B

Question 3
At Rocktown College, the 400 students taking
statistics received an average score of 76 on the
nal exam, and the scores had a normal distribution.
Students who receive a score of 66 or lower on the
exam will fail the course, and only 8 students earned
a score of 96 or higher. How many of the students
will fail the course?

146

The Princeton Review, Inc.

1, 10, 19

100, 101, 102

LESSON 5 MATH

SCRATCH PAPER

Draw a bell curve!

RESPONSE

The Princeton Review, Inc.

147

MANUAL FOR THE GRE

TRIGGER
TRIGGER

Factor, regroup, try to cancel out


common factors.

TRIGGER

Variables in the answer choices

List A,B,C,D,E on scratch paper. Plug In


(terms labeled, target number circled,
all answer choices checked).

RESPONSE

TRIGGER

Must be

Plug In more than once. Write answers


choices down the left side of scratch
paper and variables across the top. Plug
in a simple number, then use FROZEN.

RESPONSE

TRIGGER

Quant Comp with variables

Plug In more than once. Set up your


scratch paper and Plug In using
FROZEN.

RESPONSE

Urge to set up equation and do


algebra
The words how much,
how many, or what is the value of

List answer choices in a column on your


scratch paper. Label the rst column
and PITA.

RESPONSE

Exponents or numbers too big to


calculate

RESPONSE

148

Write the percentage change formula:


difference
Percent change =
100
original

RESPONSE

Question asks for percent change,


percent increase, or percent
decrease

TRIGGER

TRIGGER REVIEW

The Princeton Review, Inc.

Average

Draw an Average Pie for every time the


word average appears in the question.

RESPONSE

TRIGGER

LESSON 5 MATH

TRIGGER

Find a bunch of numbers and list them


in order on your scratch paper.

RESPONSE

TRIGGER

Ratio

Draw a ratio box on your scratch paper.

RESPONSE

Parallel lines or (

Label all big angles and small angles.

RESPONSE

1
Write down A = bh and park information
2
underneath. (The height will always be

TRIGGER

Triangle question contains the word


area

TRIGGER

Parallelogram, rectangle, or square


problem containing the word area

Write down the area formula and park


information directly underneath.

RESPONSE

TRIGGER

Perimeter

Draw the shape, label the sides, and


add the sides together.

RESPONSE

RESPONSE

Median

TRIGGER

perpendicular to the base.)

The Princeton Review, Inc.

149

TRIGGER

Circle

Write r d C A vertically on
scratch paper. Find radius, diameter,
circumference, and area.

RESPONSE

TRIGGER

Right triangle with 45 or 2, or


diagonal of a square

45-45-90 triangle.
You know its in there somewhere.

RESPONSE

TRIGGER

Right triangle with 30, 60, or 3 ,


a hypotenuse that is twice the
smallest leg, or height of equilateral
triangle

Find the 30-60-90 triangle.


You know its in there somewhere.

RESPONSE

TRIGGER

Arc or sector

Central Angle
sector
arc
=
=
360
circumference area

RESPONSE

TRIGGER

MANUAL FOR THE GRE

Group problem with overlap or


both

Write down group formula:


Total = [Group 1] + [Group 2] [Both] +
[Neither]

RESPONSE

150

The Princeton Review, Inc.

Group question in which elements


are in either one group or other (no
overlap)

Group X Group Y Total


Group A
Group B
Total

RESPONSE

TRIGGER

Draw Group Grid:

LESSON 5 MATH

SCRATCH PAPER

The Princeton Review, Inc.

151

MANUAL FOR THE GRE

DRILL
Question 1 of 15

Question 4 of 15

Quantity A

Quantity B

512 510

(23)510

Quantity A is greater.
Quantity B is greater.
The two quantities are equal.
The relationship cannot be determined
from the information given.
Question 2 of 15
A cell phone company offers two different calling
plans to its customers. With plan C, the customer is
charged $1.35 for the rst 5 minutes and 7 cents for
each additional minute. With plan D, the customer
is charged a at rate of 15 cents per minute. If x
represents the length, in minutes, of a call, what is
the greatest integer value of x for which a customer
can save money by using plan D rather than plan C?
8
9
10
11
12
Question 3 of 15
Several people rented a car for $30. If there had
been one more person in the group, it would have
cost each person $1 less. How many people were in
the group originally?

x>0
Quantity A
2
2+

152

The Princeton Review, Inc.

x
Quantity A is greater.
Quantity B is greater.
The two quantities are equal.
The relationship cannot be determined
from the information given.
Question 5 of 15
Y

Triangle XYZ in the gure above is an equilateral


triangle. If the perimeter of the triangle is 12, what is
its area?
2 3
4 3
8

5
6
10
12
15

Quantity B

12
8 3

LESSON 5 MATH

SCRATCH PAPER

The Princeton Review, Inc.

153

MANUAL FOR THE GRE

Question 6 of 15

Question 9 of 15

On a geography quiz, Emily and Katherine scored an


average of 80 points. If Katherine and Julia scored
an average of 85 points, how many more points did
Julie earn than Emily ?

Of 110 customers at a restaurant, 63 order


hamburgers and 71 order french fries. If 22
customers order neither hamburgers nor french fries,
how many customers order both hamburgers and
french fries?

15
10
7.5
5
2.5

8
30
31
46
88

Question 7 of 15
In a mixture of alcohol and water there is

more

Question 10 of 15

5
alcohol than water. What is the ratio of alcohol to
water in the mixture?
5:1
4:1
12 : 5
6:5
5:6
Question 8 of 15
List A: 3, 7, 4, 20, 10
List B: 5, 0, 8, 8, 7, 0
How much greater is the median of the numbers is
list B than the median of the numbers in list A ?

In the gure above, if MN =


circle with the center O is

2
4

154

The Princeton Review, Inc.

2 , then the area of the

LESSON 5 MATH

SCRATCH PAPER

The Princeton Review, Inc.

155

MANUAL FOR THE GRE

Question 11 of 15

Question 13 of 15
B

What is the perimeter of the isosceles triangle ABC


shown above if the measure of ABC is 120,
BD = 2 and AC = 4 3 ?

Of the 125 employees at a manufacturing company,


75 are white-collar and the rest are blue-collar. If
58 employees are male, and 24 of the female
employees are blue-collar, how many of the male
employees are white-collar?
18
23
32
43
50

4+4 3
Question 14 of 15

12

8+4 3

Fred and Bobbie are book collectors, and Fred has


twice as many books as Bobbie does. One-fourth of
Freds books are signed by the authors, and threefths of Bobbies books are signed by the authors.
If Fred and Bobbie combine their collections, what
fraction of the books are signed by the authors?

8+8 3
24
Question 12 of 15
B

C
50
F

Question 15 of 15
D

In the gure above, if BC ( EF ( DG and


ED = EA, what is the measure of DEA ?
40
50
60
65
80

20 cm

In the gure above, if the area of the larger circular


region is twice the area of the smaller circular region,
how many centimeters long is the radius of the
smaller circular region?
5
2
5
5 2
10
10 2

156

The Princeton Review, Inc.

LESSON 5 MATH

SCRATCH PAPER

The Princeton Review, Inc.

157

Lesson 5
Verbal

MANUAL FOR THE GRE

SENTENCE EQUIVALENCE REVIEW


Steps for Sentence Equivalence Questions
5

1
2.
3.
4.

Find the story.


Come up with your own word or phrase for the blank.
Check each answer choice and use your scratch paper.
POE: Yes! = No! =
Dont Know Vocab = ? Maybe =

Question 1
Despite _________ cynicism about the
commercialization of holidays such as Christmas
and Easter, few can resist getting caught up in the
spirit of the season.

.
.
.
.
.
.

guileless
widespread
lewd
pervasive
disingenuous
nave

Question 2
Many suburbanites are _________ by public
transportation as it lacks the privacy and security
afforded by a private automobile.

.
.
.
.
.
.

160

assuaged
unnerved
enervated
discomted
mollied
bemused

The Princeton Review, Inc.

SCRATCH PAPER

LESSON 5 VERBAL

Question 3

SCRATCH PAPER

Young adults are becoming _________ workers,


unlike their parents who remained at one employer
for life.

.
.
.
.
.
.

stingy
cunning
itinerant
famous
peripatetic
famous

Question 4
Once a xture on countless street corners
throughout the country, the a cappella group is now
a relative _________.

.
.
.
.
.
.

rarity
clich
nuisance
atavism
curiosity
simulacrum

Question 5
Not even the most _________ traveler can resist the
rustic charms of the small shing villages of western
Ireland.

.
.
.
.
.
.

dogged
crafty
unstinting
querulous
jaded
benevolent

The Princeton Review, Inc.

161

MANUAL FOR THE GRE

Question 6

Most often, the futurists greatest aw is not that


she has the hubris to suppose she can imagine the
technologies and social structures of tomorrow, but
that she is so myopic that she cannot _________
which traits are so indelibly human they must remain
unaltered.

.
.
.
.
.
.

discern
articulate
approximate
communicate
appreciate
propitiate

Question 7
One of the longest running series in history,
Doctor Who, a science ction _________, has thrilled
generations since 1963 with its cliff-hanging plots
and roguish main protagonist.

.
.
.
.
.
.

parody
picaresque
roman clef
fable
adventure
satire

Question 8
Unlike linguistics, which focuses on morphology and
syntax, philology also includes literature and history
within its _________.

.
.
.
.
.
.

162

ambit
purview
mythos
phoneme
lore
ontogeny

The Princeton Review, Inc.

SCRATCH PAPER

LESSON 5 VERBAL

Clues & TriggersNot Just for Sentence Equivalence!


Steps for Text Completions Questions
1
2.
3.
4.

Find the story.


Come up with your own word or phrase for the blank.
Check each answer choice and use your scratch paper.
POE: Yes! = No! =
Dont Know Vocab = ? Maybe =

Question 1

5
SCRATCH PAPER

The sudden drop in temperature, the darkening


clouds, and the increasing winds indicated that the
storm was _________.

Question 2
In the eighteenth and nineteenth centuries, the
chestnut tree ourished along the Eastern Seaboard,
but it had been almost completely _________ by the
1950s.

Question 3
Because of their _________ personalities, no one
expected their marriage to last.

Question 4
Having bestowed her patronage on many
writers and performers, not least among them
Shakespeares theater company, Queen Elizabeth I
was quite aptly labeled a grand _________ of the arts.

The Princeton Review, Inc.

163

MANUAL FOR THE GRE

Now with Answer Choices!

SCRATCH PAPER

Question 1

The sudden drop in temperature, the darkening


clouds, and the increasing winds indicated that the
storm was _________.
pervasive
controversial
irrelevant
deliberate
imminent
Question 2
In the eighteenth and nineteenth centuries, the
chestnut tree ourished along the Eastern Seaboard,
but it had been almost completely _________ by the
1950s.
cultivated
decimated
assuaged
appended
challenged
Question 3
Because of their _________ personalities, no one
expected their marriage to last.
apposite
variegated
disparate
evanescent
affable
Question 4
Having bestowed her patronage on many
writers and performers, not least among them
Shakespeares theater company, Queen Elizabeth I
was quite aptly labeled a grand _________ of the arts.
connoisseur
critic
ally
regent

164

The Princeton Review, Inc.

judge

LESSON 5 VERBAL

Multiple Blanks: Wash, Rinse, Repeat...


Question 5
The reviewer gave the new bistro a (i)________
review, claiming that while the food was innovative
and well-executed, the service and decor were
(ii)________.

SCRATCH PAPER
5

Question 6
Moby Dick, now regarded as a great work of
American literature, was virtually (i)________ when it
was rst published, and it was not until many years
later that Melvilles achievements were (ii)________.

Question 7
Pushing forward, despite hundreds of (i)__________
attempts to produce a working light bulb, Edison
eventually triumphed, his (ii)__________ contributing
to his ultimate success.

Question 8
Proponents of the International Style in architecture
called for reducing buildings to purely functional
form and found beauty in highlighting (i)_________
features. They rejected references to (ii)________
styles, offering designs indifferent to location, a
quality subsequently (iii)_________ by those who
viewed the style as bland or unappealing.

The Princeton Review, Inc.

165

MANUAL FOR THE GRE

Now with Answer Choices!


Question 5

The reviewer gave the new bistro a (i)________


review, claiming that while the food was innovative
and well executed, the service and decor were
(ii)________.
Blank (i)

Blank (ii)

glowing

inappropriate

scathing

stunning

guarded

lackluster

Question 6
Moby Dick, now regarded as a great work of
American literature, was virtually (i)________ when it
was rst published, and it was not until many years
later that Melvilles achievements were (ii)________.
Blank (i)

Blank (ii)

parodied

relegated

hailed

celebrated

ignored

merited

Question 7
Pushing forward, despite hundreds
of (i)__________ attempts to produce a working
light bulb, Edison eventually triumphed, his
(ii)__________ contributing to his ultimate success.

166

The Princeton Review, Inc.

Blank (i)

Blank (ii)

felicitous

grandiloquence

unavailing

indifference

auspicious

tenacity

SCRATCH PAPER

LESSON 5 VERBAL

SCRATCH PAPER

Question 8
Proponents of the International Style in architecture
called for reducing buildings to purely functional
form and found beauty in highlighting (i)_________
features. They rejected references to (ii)________
styles, offering designs indifferent to location, a
quality subsequently (iii)_________ by those who
viewed the style as bland or unappealing.
Blank (i)

Blank (ii)

Blank (iii)

structural

oracular

disparaged

aesthetic

provincial

embraced

hackneyed

secular

reclaimed

The Princeton Review, Inc.

167

MANUAL FOR THE GRE

Text Completions

SCRATCH PAPER

Question 1

Though it is not as high prole as a light rail or


subway, increasingly, _________ cities are turning to
a concept called bus rapid transit as an affordable
transportation option.
sprawling
impecunious
resilient
provincial
ambitious

Question 2
The television critic attributed the enduring
popularity of police and medical procedurals such
as CSI and House to the _________ desire to see
mysteries solved and diseases cured.
ephemeral
perennial
nascent
hackneyed
intermittent

Question 3
The _________ of information about the recently
discovered species meant that ornithologists had
limited familiarity with the birds mating and nesting
practices.
bounty
succor
overabundance
paucity
rubric

168

The Princeton Review, Inc.

LESSON 5 VERBAL

SCRATCH PAPER

Question 4
The staff found themselves (i)_________ by their
supervisors (ii)________ personality; his demeanor
could change from affable to irascible in the space
of a few minutes.

Blank (i)

Blank (ii)

perplexed

voracious

assuaged

priggish

sated

mercurial

Question 5
The critic (i)_________ the playwright, describing his
latest work as (ii)_________ and not illustrative of the
work of a supposedly mature talent.
Blank (i)

Blank (ii)

lambasted

exemplary

sanctioned

sophomoric

circumscribed

scrupulous

Question 6
The speaker claimed that, even though natural
disasters are (i)________ and therefore unavoidable,
much of the associated suffering could be
avoided if charitable relief organizations were to
collaborate rather than (ii)________ donations and
media attention. Though understandable, such
(iii)________ is ultimately counter-productive and in
the end makes it more likely that there would be
gaps in relief services.
Blank (i)

Blank (ii)

Blank (iii)

diabolical

vie for

obstinacy

isolated

propitiate

synergy

inexorable

rely on

rivalry

The Princeton Review, Inc.

169

MANUAL FOR THE GRE

SCRATCH PAPER

Question 7

The image of the architect as the lonely artist


drawing three-dimensional forms is (i)________ the
publics understanding of the architects role. As a
result, buildings are viewed as the singular creations
of an artistic vision (ii)________ the architect.
Certainly architects should take much of the credit
for the form of a unique building, but the nal
product is hardly a (iii)________. The architect relies
heavily upon faade consultants, engineers, and
skilled builders, while the form of the building may
depend, in addition, upon zoning regulations, cost,
and market demands.
Blank (i)

Blank (ii)

Blank (iii)

at odds with

tangentially
related to

virtuoso
performance

central to

but an
afterthought
to

collaborative
effort

irrelevant to

justiably
embodied by

physical
triumph

Question 8
To mistake (i)__________ for (ii)__________is to
confuse braggadocio for skill.

170

The Princeton Review, Inc.

Blank (i)

Blank (ii)

hubris

denouement

mendacity

satire

hedonism

competence

LESSON 5 VERBAL

Relationships Between the Blanks

It would be (i)________ for any ambitious politician to


(ii)________ such a powerful group of voters.

SCRATCH PAPER
5

Question 1
Elizabeth was (i)________ worker and therefore
expected (ii)________ of her work.
Blank (i)

Blank (ii)

a diligent

ignorance

a listless

dismissal

an insolent

approbation

Question 2
Since the producer was (i)________ the staggering
costs typically associated with post-production
editing, she was surprised when she saw the
(ii)________ amount allocated to that line item in her
editors budget.
Blank (i)

Blank (ii)

unaware of

scant

cognizant of

unintelligible

apathetic to

apposite

The Princeton Review, Inc.

171

MANUAL FOR THE GRE

READING COMPREHENSION

Line
5

10

15

20

25

30

35

40

45

172

The Princeton Review, Inc.

In the past 10,000 years, biological diversity


has entered a new era. Humans have adversely
affected species diversity; the rate of humaninduced extinctions is accelerating. The careless
introduction of the Nile perch has threatened
hundreds of sh species native to Lake Victoria.
Decimated forests and excessive hunting have
eliminated half of the bird species of Polynesia.
The list of comparable disasters is substantial.
The history of global diversity, summarily
explained, initiated after the incipient proliferation of multi-cellular animals. This gave rise to
a precipitous increase in the number of species
in early Paleozoic times (between 600 and 430
million years ago) followed by stagnation for the
remaining 200 million years of the Paleozoic era,
and ultimately a slow but steady climb through
the Mesozoic and Cenozoic eras until biological
diversity reached its apex. Such a long and storied history intimates that this heterogeneity was
a hard-fought battle.
The human species emerged at this seemingly
optimal time of extreme biological diversity. Yet
today, as human populations swell, they transform their environments, reducing biological
diversity to its lowest level, a level not seen
since the end of the Mesozoic era, 65 million
years ago. Final consequences of this devastation are beyond calculation; some have termed
this The Biodiversity Crisis.
Unlike the quotidian calculations of material
and cultural wealth, assessments of biological
wealth are almost non-existent and and thus
usually taken for granted, an egregious and
catastrophic error. Every species is unique and
irreplaceable, making the loss of biodiversity the
most profound process of environmental change.
Furthermore, its consequences are also the least
predictable since the Earths biotathe fauna
and ora collectivelyremains largely unstudied
and unappreciated. It took millions of years of
evolution to engender such diverse biota; drastic
change to this equilibrium cannot be anything
but disastrous. Moreover, the Earths biota also
presents a potential source for massive untapped
material wealth in the form of food, medicine,
and other commercially important substances
resources that are essential for the ever-expanding human species.

LESSON 5 VERBAL

Question 1
The passage is primarily concerned with
cautioning against the massive extinction
of numerous species that would make past
extinctions look minor by comparison
tracing the evolution of biodiversity to
contrast with that of humans
explaining the origins of diversity of
life on Earth to warn against conferring
preeminence on one species over others
warning against irreversibly reducing
the biodiversity of the planet, thereby
representing a momentary setback both for
science and for humanity as a whole
describing the current decline in species
diversity precipitated by humans as a
tragedy of incalculable proportions that
could have dire consequences

SCRATCH PAPER

Question 2
Consider each of the choices separately and select
all that apply.
Which of the following are explicitly mentioned in
the passage as contributing to the extinction of
species?

. Hunting
. Polluting
. Human-induced changes in the
environment
Question 3
The author would most likely agree with which of the
following statements about biodiversity?
There is a precise and optimal level of
biodiversity that should be maintained.
The current level of biodiversity is unique.
The loss of biodiversity is the most perilous
predicament to befall the inhabitants of
Earth.
The value of biodiversity is often
overlooked.
Biodiversity reached its peak during the
Paleozoic era.

The Princeton Review, Inc.

173

MANUAL FOR THE GRE

Line
5

5
10

15

20

25

30

35

40

174

The Princeton Review, Inc.

The tropical leaf-cutter, or attine, ant provides


a remarkable example of complex symbiosis
involving several species. The ants cultivate a
fungus in underground caverns to serve as a
source of food. DNA analysis reveals that the
fungi in attine gardens around the world are
clones of a single source. The ants do not allow
their fungus crop to develop fruiting bodies, the
means by which plants engage in sexual reproduction. Instead, a queen ant starting a new nest
takes a sample of the fungus from the old nest
to start the new garden, spreading the fungus
vegetatively, or asexually.
For many years, the phenomenon of sexual
reproduction puzzled biologists, as it passes only
half of the parents genes to the succeeding generation and requires a more complex mechanism
than does asexual reproduction. What benet
of sexual reproduction would outweigh these
limitations? One likely answer is that it provides
a defense against parasitic attack. Simple parasites, such as bacteria or molds, mutate rapidly
and pose a challenge to other species. The rapid
evolution of the attacking species provides an
advantage in this biological arms race. Sexual
reproduction allows the more complex species
to shufe its genes between generations and,
as a result, evolve quickly enough to match the
parasites mutations.
A clonal monoculture, such as that maintained
by the attine, should be highly susceptible to
parasites, yet preliminary research suggested no
evidence of such a problem in the ants gardens.
More comprehensive studies showed that the
Escovopsis mold, a parasite related to the green
mold known to commercial mushroom farmers, is present in the ants crops and poses a
serious threat to the fungus. However, the attine
ants provide the defense mechanism lacking
in the funguss asexual reproduction by means
of a bacterium that grows in patches on their
skin. This actinomycete bacterium produces an
antibiotic used to control the mold and limit its
destructive effects on the ants food source.

LESSON 5 VERBAL

Question 1
Which of the following can be most reasonably
inferred from the passage?

SCRATCH PAPER

 The fruiting bodies found on numerous


fungi are a means of vegetative
reproduction.
 Plants that rely on sexual reproduction are
highly susceptible to parasitic attack by
bacteria and molds.
 Sexual reproduction is an inefcient
method for transmitting a parents genes
to its offspring.
 The bacterium found on patches of green
mold produces an antibiotic substance
used by commercial mushroom farmers.
 Parasitic bacteria use sexual reproduction
as a means to achieve rapid mutation.

Question 2
The author describes the interaction between a
parasite and a sexually reproducing organism as an
arms race (line 25) in order to

 emphasize the aggressive nature of the


parasitic organism
 warn of the dangers of biological weapons
 underscore the need for the attine ants to
defend their gardens
 point out the devastating effects of the
Escovopsis mold
 depict the result of rapid evolution by both
organisms
Question 3
The passage suggests which of the following about
the fungus grown by the attine ants?

 By shufing its genes, it is able to mutate


rapidly in response to parasitic threats.
 Due to its asexual reproduction, it is
susceptible to attack by the actinomycete
bacterium.
 It could potentially engage in sexual
reproduction.
 It is closely related to the crops grown by
commercial mushroom farmers.
 Without the care of the attine ants, it would
quickly become extinct.
The Princeton Review, Inc.

175

MANUAL FOR THE GRE

TEXT COMPLETIONS SUMMARY


Steps for Text Completions Questions

176

The Princeton Review, Inc.

1.

Find the story.

2.

Come up with your own word or phrase for the


blank.
Look for Clues and Triggers. If youre still
stumped, try finding a Relationship Between the
Blanks.

3.

Check each answer choice and use your scratch paper.

4.

POE: Yes! =
Maybe =

No! =

Dont Know Vocab = ?

Lesson 6
Math

MANUAL FOR THE GRE

Question 1
A piggy bank holds exactly 6 dimes and 10 nickels.
Four coins are selected at random from the bank,
one at a time, without replacing the coins. If the rst
3 coins selected are all nickels, what is the
probability that the next coin selected will also be a
nickel?

TRIGGER

16

13

13

13

Trigger: _______________________________________________________________

Question 2
75 colored tiles are placed in a bag.

of the tiles are


3
blue, and 20% of the remaining tiles are green. If all

the other tiles are white, what is the probability that


a tile selected at random will be a white tile?
Give your answer as a fraction.

178

The Princeton Review, Inc.

LESSON 6 MATH

SCRATCH PAPER

want
total

RESPONSE

Draw a line for each event and nd the total.

The Princeton Review, Inc.

179

MANUAL FOR THE GRE

Question 3

A questionnaire was sent to all 40 employees of a


certain company to determine if they were available
for a meeting on either Monday or Wednesday.
Eight employees were unavailable on either day, 20
were available on Monday, and 17 were available on
Wednesday. If an employee is selected at random,
what is the probability that he or she is available
both Monday and Wednesday?
Give your answer as a fraction.

Question 4
Grifn has a bag of marbles that contains only six
black marbles and four red marbles. If he removes
three marbles at random without replacing any of
the marbles, what is the probability that all three
marbles selected will be red?
2

25

125

30

Question 5
Alejandro ips a coin four times. What is the
probability that he will get heads on the rst 2
throws and tails on the last 2 throws?
1

16

Question 6
The probability that event A will occur is 0.57.
Quantity A
0.61

180

The Princeton Review, Inc.

Quantity B
The probability that
both event A and
event B will occur

LESSON 6 MATH

SCRATCH PAPER
To nd the probability of multiple events happening in a row,
multiply the probability of each event.
Probability of A and B = Probability of A Probability of B

The Princeton Review, Inc.

181

MANUAL FOR THE GRE

Question 7
Ann has a deck of 52 cards, made up of four suits,
each with cards numbered 1 through 13. If she
selects a card at random, what is the probability that
she selects a 2 or a 7 or a 9 ?

3,825

64

52

13

Question 8
1
The probability of rain on each of ve days is ,
6
2
except on the rst day, when it is , and on the last
5
4
day when it is . What is the probability that rain
5
will occur on at least one of the ve days?
1

22

67

675

72

27

27

72

Question 9
Four people are playing a game in which each
person rolls a six-sided die at the same time.
Quantity A
2
3

182

The Princeton Review, Inc.

Quantity B
The probability that
at least two of the
people roll the same
number

LESSON 6 MATH

SCRATCH PAPER
On the GRE, to nd the probability that any one of several
different events happens, add the probability of each event.
Probability of A or B = Probability of A + Probability of B

To nd the probability for NOT or at least questions,


nd the probability that the event does not happen, and subtract
it from 1.
Probability of A = 1 Probability of not A

The Princeton Review, Inc.

183

MANUAL FOR THE GRE

Question 10
Points A and C lie on the circle (not shown) with
center O such that AOC is equilateral. What is the
probability that a randomly selected point on the
circle lies on minor arc AC ?
1

360

60

6
Question 11

C
5

Note: Figure not drawn to scale.


In the gure above, ABCD is a rectangle. If BC = 15,
what is the probability that a randomly chosen point
that lies in ABCD lies inside the shaded region?
1

15

10

10

Question 12
B

If rectangle ABCD shown above has an area of 24,


what is the probability that a randomly chosen point
that lies in ABCD will lie in the shaded region?

184

The Princeton Review, Inc.

It cannot be determined
from the information
given.

LESSON 6 MATH

SCRATCH PAPER
For probability with questions involving area,
use the following formula: area you want .
total area

The Princeton Review, Inc.

185

MANUAL FOR THE GRE

Question 1
x2 + 9x + 20 = 0
Quantity A
x

Quantity B
3

TRIGGER

Trigger: _______________________________________________________________

Question 2
If x2 + 19x + 48 = 0, then which of the following is a
possible value of x ?
Indicate all such values.

.
.
.
.
.
.
.
.

2
3
6
8
12
16
24
48

Question 3
x2 + 15x + 56 = 0
If x satises the equation above, what is the greatest
possible value of x ?

186

The Princeton Review, Inc.

LESSON 6 MATH

SCRATCH PAPER

Put the equation in the form x2 + bx + c = 0


)(x

)=0

Remember A.M.
Add
x2 + bx

Multiply
+c = 0

Find two numbers that multiply to c and add to b. If necessary, write out
every factor of c.

RESPONSE

Underneath equation, write down (x

The Princeton Review, Inc.

187

MANUAL FOR THE GRE

Question 4
x2 12x 13 = 0
Quantity A
x






Quantity B
2

Quantity A is greater.
Quantity B is greater.
The two quantities are equal.
The relationship cannot be determined from the
information given.

Question 5
What is the least possible value of x which satises
the equation x2 + 5x 6 ?

Question 6
a2 + b2 = 16
(a + b)2 = 30
Given the equations above, what is the value of ab ?

188

The Princeton Review, Inc.

LESSON 6 MATH

SCRATCH PAPER

The Princeton Review, Inc.

189

MANUAL FOR THE GRE

Question 7
If x is an integer such that x2 = 3x + 10, then x could
be which of the following?

 10
 5
 2
 5
 There is no such value of x.

Question 8
( 7 + 2 ( ( 5 1( ( 5 + 1( ( 7 2( =

Question 9
If a = x2 + 2x, and a = 15, which of the following is a
possible value for x ?
Indicate all such values.

. 13
. 5
. 3
.
3
.
5
. 13

190

The Princeton Review, Inc.

LESSON 6 MATH

SCRATCH PAPER

The Princeton Review, Inc.

191

MANUAL FOR THE GRE

Question 10
In the equation x2 + px 18 = 0, one of the roots is 6,
and p is a constant. What is the value of p ?

 3
 1
 3
 6
 18

Question 11
One of the factors of the equation x2 + 9x + c is
(x + 11), where c is a constant. Which of the
following is one possible value of x ?

 5
 2
 2
 5
 11

Question 12
x2 + kx + 12 = 0
y2 + ky 30 = 0
In the above equations, k is a constant. What is the
value of k ?

192

The Princeton Review, Inc.

LESSON 6 MATH

SCRATCH PAPER

The Princeton Review, Inc.

193

MANUAL FOR THE GRE

DRILL
Question 1 of 10
0 g5
0 h1
Which of the following could be the value of g h ?

Select all such values.

.
.
.
.
.
.
.

1
0
1
2
3
4
5

Question 2 of 10
There are four cookies in a jar: chocolate chip,
peanut butter, sugar, and oatmeal. If two are chosen
at random, what is the probability that one of the
two cookies will be chocolate chip?

12

Question 3 of 10
Jordan is j years old. Five years ago, Marshall was
three times older than Jordan will be in two years. In
terms of j, how old is Marshall now?




j
3
j

11
5

 3j + 5
 3j + 11
j+7

194

The Princeton Review, Inc.

LESSON 6 MATH

SCRATCH PAPER

The Princeton Review, Inc.

195

MANUAL FOR THE GRE

Question 4 of 10
In a certain dice game, two six-sided dice are rolled.
A player scores a single point if and only if he rolls
a 1 or a 5 on at least one of the dice. On any given
roll, what are the chances that a player will score a
point?

Question 5 of 10
ab 0
Quantity A
(3a 3b)

Quantity B
9a2 + 9b2

Question 6 of 10
Debbi is voting for three different ofces in a
statewide election. If her chosen candidate for
governor has an 60% chance of winning, her chosen
candidate for senator has a 75% chance of winning,
and her chosen candidate for county supervisor has
a 37.5% chance of winning, what is the probability
that at least one of Debbis chosen candidates will
win the election?

27

123

133

15

16

160

160

160

16

Question 7 of 10
x2 36 and y2 9
Quantity A
x
y
196

The Princeton Review, Inc.

Quantity B
2

LESSON 6 MATH

SCRATCH PAPER

The Princeton Review, Inc.

197

MANUAL FOR THE GRE

Question 8 of 10
Points A, B, and C, lie on the circumference of a
circle with center O. If the circle has a diameter of
2 and arc BCA has a length of

, what is the least


30
possible measure, in degrees, of angle BOA?

Question 9 of 10
(q + 5)(q 7) = 0
Quantity A
2

q 2q 35






Quantity B
q

Quantity A is greater.
Quantity B is greater.
The two quantities are equal.
The relationship cannot be determined from the
information given.

Question 10 of 10
Quantity A

Quantity B

(y + 4) 8y

y2 + 15






198

The Princeton Review, Inc.

Quantity A is greater.
Quantity B is greater.
The two quantities are equal.
The relationship cannot be determined from the
information given.

LESSON 6 MATH

SCRATCH PAPER

The Princeton Review, Inc.

199

MANUAL FOR THE GRE

SUMMARY

TRIGGER

x and x 2 in the same equation


or anything that looks like a
quadratic

Underneath the equation, write


down (x ) (x ) = 0.
Remember A.M.

RESPONSE

Put the equation in the form


x + bx + c = 0.

200

The Princeton Review, Inc.

Draw a space for each event.


probability
want

Find the total(s). total

RESPONSE

TRIGGER

If necessary, nd every factor


of c.

Lesson 6
Verbal

MANUAL FOR THE GRE

COMMON FLAWS
Ninety percent of the population of Prelandia
lived in rural areas in 1800. Today, only 20
percent of the population lives in rural areas.
Clearly, more people lived in the countryside two
centuries ago.

6
Contrary to popular belief, high school
students overwhelmingly approve of the high
school administrative staff. We know this to
be true because the student council expressed
admiration for the high school principal and
her staff in the councils editorial for the school
paper.

Motorboating accidents and ice cream sales


increase at the same time of year. Therefore,
increased consumption of ice cream causes
motorboating accidents.

New York City has a mass transit system


which relies upon buses, subways, and light rail.
Ogdenville, a town with a population of 30,000,
wishes to overhaul its mass transit system. Thus,
Ogdenville should build a mass transit system
which relies upon buses, subways, and light rail.

202

The Princeton Review, Inc.

LESSON 6 VERBAL

SCRATCH PAPER
Type of Flaw: ______________________________________________________________________
__________________________________________________________________________________
How to strengthen: __________________________________________________________________
__________________________________________________________________________________
How to weaken: ____________________________________________________________________
__________________________________________________________________________________

Type of Flaw: ______________________________________________________________________


__________________________________________________________________________________
How to strengthen: __________________________________________________________________
__________________________________________________________________________________
How to weaken: ____________________________________________________________________
__________________________________________________________________________________

Type of Flaw: ______________________________________________________________________


__________________________________________________________________________________
How to strengthen: __________________________________________________________________
__________________________________________________________________________________
How to weaken: ____________________________________________________________________
__________________________________________________________________________________

Type of Flaw: ______________________________________________________________________


__________________________________________________________________________________
How to strengthen: __________________________________________________________________
__________________________________________________________________________________
How to weaken: ____________________________________________________________________
__________________________________________________________________________________

The Princeton Review, Inc.

203

MANUAL FOR THE GRE

Question 1
The following appeared in an email from the
president of CheapSteak Foods.

Six months ago, we opened our rst restaurant,


CheapSteak, which sells quickly-prepared and
low-priced steak sandwiches. Waitstaff have
reported that the price and convenience drew
many customers away from nearby restaurants,
which are generally more expensive. Although
many of these restaurants have since tried to
compete with CheapSteak by lowering their prices
and adding drive-thru windows, CheapSteak has
made a prot every month since it opened. Given
our success with our rst restaurant, we believe
it is time to open another CheapSteak restaurant
and launch a frozen food line.
Question 2
The following appeared in a memo to the head of
marketing for Dulcied Treacle Co., an established
candy company.
In the last four years, gross sales in the candy
market have remained static, but ice cream,
another confectionary product, has experienced
huge increases. Specically, the growth of
boutique ice cream brands specializing in
unusual, savory ice cream avors such as pink
peppercorn, basil, and ginger, has exploded. In
response, we gave out free samples of our new
savory-avored candy chews at a number of
national gourmet food fairs, and in every case,
our supplies ran out within minutes. Therefore,
we should jump to the forefront of this trend and
launch our savory candy chews nationally at all
of our retail outlets.
Question 3
The following appeared in a letter from a car owner
to a business associate.

204

The Princeton Review, Inc.

Of the two leading used car lots in our town


Lyme Cars and Limmon AutomobilesLyme
Cars is clearly better. Lyme Cars has more than
a square mile of cars to choose from, whereas
Limmon Automobiles has a much smaller
indoor showroom, with fewer cars to choose
from. Lyme also has twelve sales associates; in
contrast, Limmon only has four, two of whom
work only part-time. Cars from Lyme are better
as well: Seven years ago, I purchased a car from
Limmon for $7,000, and it broke down within a
year. A car I purchased from Lyme Cars last year
cost only $4,000 and is still running smoothly.
Thus, if you want to buy an inexpensive, quality
used car, you should use Lyme Cars.

LESSON 6 VERBAL

Conclusion: _______________________________________________________________________
Premise: __________________________________________________________________________
__________________________________________________________________________________
Assumptions: ______________________________________________________________________
__________________________________________________________________________________
__________________________________________________________________________________

__________________________________________________________________________________
__________________________________________________________________________________

Conclusion: _______________________________________________________________________
Premise: __________________________________________________________________________
__________________________________________________________________________________
Assumptions: ______________________________________________________________________
__________________________________________________________________________________
__________________________________________________________________________________
__________________________________________________________________________________
__________________________________________________________________________________

Conclusion: _______________________________________________________________________
Premise: __________________________________________________________________________
__________________________________________________________________________________
Assumptions: ______________________________________________________________________
__________________________________________________________________________________
__________________________________________________________________________________
__________________________________________________________________________________
__________________________________________________________________________________

The Princeton Review, Inc.

205

MANUAL FOR THE GRE

An ancient, traditional treatment to help reduce


obesitycoconut oilhas recently been proven
effective. In a study completed last year, 50
volunteers who had issues maintaining a healthy
weight were fed a special diet rich in coconut oil for
two weeks. No volunteer was allowed to eat outside
foods during the trial, and each volunteer had his
or her weight measured several times each day.
After 2 weeks, the volunteers had lost an average
of 8 pounds each. Therefore, the study proves that
coconut oil reduces obesity within a short period of
time.

Write a response in which you discuss one or more alternative explanations that could rival the proposed
explanation and explain how your explanation(s) could plausibly account for the facts presented in the
argument.
Conclusion: _______________________________________________________________________
Premise: __________________________________________________________________________
__________________________________________________________________________________
Assumptions: ______________________________________________________________________
__________________________________________________________________________________
__________________________________________________________________________________
__________________________________________________________________________________
__________________________________________________________________________________
__________________________________________________________________________________
__________________________________________________________________________________
__________________________________________________________________________________

Essay Question Types

206

What are the stated or unstated assumptions of this argument?


What questions would need to be answered to determine whether the recommendation is
valid?
What evidence is needed to evaluate the argument?
What other explanations could rival the proposed explanation?

The Princeton Review, Inc.

LESSON 6 VERBAL

SCRATCH PAPER
Introduction: _______________________________________________________________________
__________________________________________________________________________________
__________________________________________________________________________________
__________________________________________________________________________________

Flaw #1

What is the assumption: ______________________________________________________________


__________________________________________________________________________________
Why that is a problem: _______________________________________________________________
__________________________________________________________________________________
How to fix it: _______________________________________________________________________
__________________________________________________________________________________

Flaw #2
What is the assumption: ______________________________________________________________
__________________________________________________________________________________
Why that is a problem: _______________________________________________________________
__________________________________________________________________________________
How to fix it: _______________________________________________________________________
__________________________________________________________________________________

Flaw #3 (optional)
What is the assumption: ______________________________________________________________
__________________________________________________________________________________
Why that is a problem: _______________________________________________________________
__________________________________________________________________________________
How to fix it: _______________________________________________________________________
__________________________________________________________________________________
Conclusion: ________________________________________________________________________
__________________________________________________________________________________
__________________________________________________________________________________
The Princeton Review, Inc.

207

MANUAL FOR THE GRE

ARGUMENT QUESTIONS
1.
2.
3.
4.

Identify the question and set up your scratch paper.


Dissect the argument.
Predict what the answer should do.
Use POE to find the answer.

Question 1
Many people predicted that business at Jacks
Bistro, previously the only restaurant in Santa
Monica, would suffer once its only competitor,
Cindys Cafe, opened across the street last year. Yet,
the average number of lunches served at Jacks
Bistro has actually increased signicantly in the past
12 months.
Which of the following, if true, most helps to explain
the increase?
Unlike Jacks Bistro, Cindys Caf serves
considerably more meals on weekends
than it does on weekdays.
Jacks Bistro has hired away a signicant
proportion of the staff who formerly
worked at Cindys Cafe.
The prot per meal is higher, on average,
for meals served at Jacks Bistro than for
those served at Cindys Cafe.
The number of restaurant patrons in Santa
Monica has doubled in the past 18 months.
Most of Jacks Bistros patrons had
never dined in Santa Monica before this
restaurant opened.

208

The Princeton Review, Inc.

Question 2
A recent study revealed that participants who
overate also produced high concentrations of
the hormone ghrelin while patients who did not
overeat had normal or low concentrations of this
hormone. Clearly, ghrelin is at least partly responsible for overeating.
The argument assumes which of the following?
Overeating does not affect the production
of ghrelin.
No other hormone affects overeating.
No other hormone is as responsible for
overeating as is ghrelin.
Ghrelin is a more important causative factor
for overeating than willpower.
People with low or normal concentrations
of ghrelin will never overeat.

LESSON 6 VERBAL

SCRATCH PAPER

The Princeton Review, Inc.

209

MANUAL FOR THE GRE

Question 3

Music industry executives have recently expressed


concern that the easy access to free digital
recordings will dampen the willingness of customers
to purchase music, but this concern is unwarranted.
As recorded music became widely available in
the form of LPs, and later cassette tapes and
CDs, people continued to attend concerts and live
music venues. Given that other recorded formats
did not cause people to stop paying for musical
entertainment, it is unlikely that the availability of
digital recordings will encourage customers to cease
making music purchases.
Which of the following, if true, most seriously
undermines the argument?
The quality of free digital recordings varies
widely depending on their source.
People value the social interaction provided
by concerts and live music venues.
A substantial number of music acionados
are intrigued by the art on CD covers,
and such art is not provided with digital
recordings.
People who attend concerts frequently are
more likely than others both to purchase
music and to download free recordings.
Some of the most popular forms of music
cannot be released in a digital format.

210

The Princeton Review, Inc.

Question 4
Although computer manufacturer X has
experienced decreasing sales in the last quarter,
stockholders should not sell their shares of
the company. The stock price of a nancially
troubled company can rise dramatically once
those problems are solved. Last year, the stock
price of steel manufacturer Y rebounded after
the company reduced its accounts receivable
backlog.
The bolded phrases play which of the following roles
in the argument above?
The rst phrase contains the authors
conclusion and the second phrase contains
unrelated information.
The rst phrase states a position and
the second phrase provides evidence to
undermine that position.
The rst phrase states a premise on which
the conclusion is based and the second
phrase provides the conclusion.
The rst phrase states the conclusion and
the second phrase supports the conclusion
with an analogy.
The rst phrase offers advice and the
second phrase draws a contrast between
two companies.

LESSON 6 VERBAL

SCRATCH PAPER

The Princeton Review, Inc.

211

MANUAL FOR THE GRE

Question 5

Question 6

Excessive government regulation, not the current high price of oil, is responsible for the poor
industrial production in Country A since its new
government came to power. Country B pays the
same price for oil, but while industrial output in
Country A has been falling, it has been rising in
Country B.

In the past ve years, there has been a marked


decline in the consumption of soft drinks sweetened with sugar. During the same time, there
has been increasing publicity about the adverse
health effects of weight gain caused by the
consumption of sugar-laden drinks. Clearly, the
publics awareness of the harmful health effects
of weight gained by consuming sugar-laden
drinks caused the decline in the consumption of
such drinks.

Which of the following, if true, would cast the most


doubt on the argument above?
Agricultural production is also declining in
Country A.
Whereas Country B must import most
minerals, Country A exports minerals.
The credit markets in both Country A and
Country B have been tight.
The industrial goods produced in Country A
are different from those that are produced
in Country B.
Country As new government increased
regulation of its industry to promote a
cleaner environment.

212

The Princeton Review, Inc.

Which of the following, if true, most stengthens the


argument above?
Many consumers have switched to drinking
diet soft drinks which are not sweetened
with sugar.
The average persons consumption of
soft drinks sweetened with sugar has
decreased by 8.2 ounces per day.
Consuming large quantities of sugarladen drinks has also been linked to mood
swings throughout the day.
The consumption of packaged cookies
which feature sugar as a key ingredient
has decreased during the time period in
question.
The price of sugar-laden soft drinks has
remained stable for the past ve years.

LESSON 6 VERBAL

SCRATCH PAPER

The Princeton Review, Inc.

213

MANUAL FOR THE GRE

Question 7
Poppy: High taxes have a chilling effect on the
economy. When individuals and corporations are
taxed, they have less money to spend. Demand
for products and services is reduced, causing unemployment to increase. Taxes must be lowered.

Lilly: But you must also consider that taxes


generate funds for the government. If taxes are
lowered, the government will be forced to borrow more money, thus reducing the amount of
credit available. Unable to borrow money easily,
businesses and individuals will be forced to limit
their purchases.
Lilly objects to Poppys argument by
claiming that Poppy has exaggerated the
adverse effects of high taxes
indicating that Poppy has based his
argument on insufcient evidence about
the effects of taxes on the economy
noting that Poppy has failed to adequately
dene the term taxes
demonstrating that danger of reducing
taxes is far more severe than the threat of
maintaining them at current levels
suggesting that the economic benets
of easy credit outweigh the danger of
unemployement

214

The Princeton Review, Inc.

LESSON 6 VERBAL

SCRATCH PAPER

The Princeton Review, Inc.

215

Lesson 7
Math

MANUAL FOR THE GRE

COORDINATE GEOMETRY
Question 1

y
Q (4, 5)

P (1, 1)

x
In the gure above, what is the length of PQ ?
5
6
4 3
7

TRIGGER

5 2

Trigger: _______________________________________________________________

Question 2
A courier is making a series of deliveries. First, she
drives 3 miles due west. Then, she turns and drives
6 miles due south. Finally, she turns and drives
5 miles due west. What is the distance between her
starting and ending locations?

218

The Princeton Review, Inc.

LESSON 7 MATH

SCRATCH PAPER

RESPONSE

Draw triangles and look for Pythagorean triples.

The Princeton Review, Inc.

219

MANUAL FOR THE GRE

Question 3
Coworkers Max and Selina are both driving to their
respective homes from work. Max drives west at
30 miles per hour for 20 minutes and then north at
60 miles per hour for 3 minutes. Selina drives east
at 30 miles per hour for 4 minutes and then south at
40 miles per hour for 3 minutes. What is the straightline distance in miles between Maxs home and
Selinas home?

Question 4

y
A
G

B
C

In the gure above, G, D, and C are points on the


x-axis, A, D, and E are points on the y-axis, and
squares ABCD and DEFG have areas of 16 and
25, respectively. What is the length of line BF
(not shown) ?
9
9 2
10 3
20 2
41

220

The Princeton Review, Inc.

LESSON 7 MATH

SCRATCH PAPER

The Princeton Review, Inc.

221

MANUAL FOR THE GRE

SLOPE
Question 1
Line k passes through points A and B, the coordinates
of which are (2, 3) and (5, 10), respectively. What is the
slope of line k ?

TRIGGER

Trigger: _______________________________________________________________

Question 2
Points C and D are located at (3, 4) and (5, 12)
respectively. Line m passes through points C and D.
What is the slope of a line that is perpendicular to
line m ?

 4
 2




222

The Princeton Review, Inc.

1
2
1
2
2

LESSON 7 MATH

SCRATCH PAPER

RESPONSE

vertical change y Rise y 2 y 1


=
=
=
horizontal change x Run x x

The Princeton Review, Inc.

223

MANUAL FOR THE GRE

EQUATION OF A LINE
Question 1
What is the x-intercept of the line dened by the
equation y = 2x + 3 ?

2 2
,
3 3

3
0,
2

3
, 0
2
(2, 0)

TRIGGER

(0, 3)

224

The Princeton Review, Inc.

Trigger: _______________________________________________________________

LESSON 7 MATH

SCRATCH PAPER

RESPONSE

Write y = mx + b on your scratch paper. Park the information you have


directly underneath.

If you know two points, you can nd the slope.


If you know a point and the slope, you can nd any other point on
the line.

The Princeton Review, Inc.

225

MANUAL FOR THE GRE

Question 2

(p, q)
(8, 6)

In the rectangular coordinate system above, q = 9.


If the line passes through the origin, O, what is the
value of p ?
6

12

Question 3
y

The equation of the line graphed on the


rectangular coordinate system above is:

y=

10 x

+2

11
Quantity A
GO






226

The Princeton Review, Inc.

Quantity B
HO

Quantity A is greater.
Quantity B is greater.
The two quantities are equal.
The relationship cannot be determined from the
information given.

13

LESSON 7 MATH

SCRATCH PAPER

The Princeton Review, Inc.

227

MANUAL FOR THE GRE

ARRANGEMENTS & COMBINATIONS


Question 1
A space shuttle crew has a Commander, a Medical
Ofcer, and a Chief Scientist. There are 5 candidates
for Commander, 4 for Medical Ofcer, and 3 for
Chief Scientist. If no candidate applies for more than
one position, then how many different shuttle crews
are possible?

Question 2
There are ve runners in a race.
Quantity A
10






228

The Princeton Review, Inc.

Quantity B
The total number of
possible arrangements
for the runners from rst
place to third place

Quantity A is greater.
Quantity B is greater.
The two quantities are equal.
The relationship cannot be determined from the
information given.

LESSON 7 MATH

SCRATCH PAPER
Arrangements & Combinations
1. Write dashes for each spot.
2. Label any restrictions.
3. How many options can go in each spot?
4. Does order matter?

The Princeton Review, Inc.

229

MANUAL FOR THE GRE

Question 3
Six children, one boy and ve girls, must stand in a
line. If the boy cannot stand rst or last in line, how
many different ways could the children be arranged?
720
480
360
240
120

Question 4
A congressional committee on legislative
procedures is to be made up of four members.
If 10 representatives are available, how many
different ways are there to make up the fourmember committee?
60
120
210
400
720

230

The Princeton Review, Inc.

LESSON 7 MATH

SCRATCH PAPER

The Princeton Review, Inc.

231

MANUAL FOR THE GRE

ARRANGEMENTS & COMBINATIONS DRILL


Question 1 of 5
Beth has ve trophies. She wants to put them on
her mantle from left to right. How many different
arrangements of trophies can she create on the
mantle?
20
60
120
176
216

Question 2 of 5
On Random Omelet Monday, a chef creates omelets
by randomly choosing three out of a possible six
llings. How many different omelets can the chef
possibly make?
9
18
20
120
720

Question 3 of 5
Grifth Bank & Trust assigns each of its depositors
a 4-digit personal identication number (PIN). What
is the probability that a randomly generated PIN will
have no repeated digits?

232

The Princeton Review, Inc.

LESSON 7 MATH

SCRATCH PAPER

The Princeton Review, Inc.

233

MANUAL FOR THE GRE

Question 4 of 5
A sundae may be ordered with any of seven
possible toppings.
Quantity A
The number of
different ways to order
a sundae with two
toppings






Quantity B
The number of
different ways to order
a sundae with ve
toppings

Quantity A is greater.
Quantity B is greater.
The two quantities are equal.
The relationship cannot be determined from the
information given.

Question 5 of 5
The co-op board of a certain residential building
must consist of two men and three women. If there
are six men and seven women who want to be on
the committee, how many different make-ups of the
committee exist?


65

525
 1,050
 1,287
 100,800

234

The Princeton Review, Inc.

LESSON 7 MATH

SCRATCH PAPER

The Princeton Review, Inc.

235

Lesson 7 Verbal

MANUAL FOR THE GRE

Line
5

10

7
15

20

25

30

35

40

45

50

238

Initially the Gramm-Leach-Bliley Actalso


known as the Financial Services Modernization
Act of 1999which repealed parts of the GlassSteagall Act of 1933 and of the Bank Holding
Company Act of 1956, both thought to be vital to
controlling egregious speculative investing and
preventing oligopolistic control of the nancial
sector, was passed to streamline the nancial
services industry by allowing any nancial institution to act as any combination of an investment
bank, commercial bank, and insurance company.
The Gramm-Leach-Bliley Act (GLB) efciently
allowed the consumer to save and invest with
the same institution. This had the potential to
minimize the distress felt by nancial institutions
when banking customers react to changes in the
economy. In prosperous times, people invest, but
in times of privation people save. GBL allowed a
nancial institution to provide both services to its
customers thereby allowing it to prosper in both
economically good times and bad. The primary
intent of the legislation was to ensure a safer
nancial sector, a benet not just for the sector
itself but also for every consumer of nancial
services.
Some consumer advocates argued however,
that such protection for investment banks came
at the expense of consumer protection from socalled mega-banksbanks that were the result
of mergers among commercial banks, insurance
companies, and investment banks. This response
may be motivated in part by the natural reluctance to repeal government regulation that had
been enacted as a safeguard. Critics go on to
point out that, even now, after lengthy consideration of the acts admirable and legitimate aims,
one cannot avoid the conclusion that the harms
engendered by the removal of safeguards, preventing both oligopoly and the malfeasance that
would surely result from allowing those banks
that control credit (lending) to also control the
use of credit (investing) outweigh the benets
of added security for individual nancial institutions. However laudable the aims of the act and
whatever impact it might have on consumers,
its critics overlook the acts greatest weakness:
its overreliance upon a fallacious assumption;
namely, that ensuring the security of individual
banks will ensure the security of the banking
system as a whole.

The Princeton Review, Inc.

Question 1
The primary purpose of the passage is to
discuss the implications of a legislative act

 summarize a historical change in regulatory


principles
 propose guidelines for future, corrective
legislation
 stipulate conditions for acceptance of
corrective legislation
 defend a corrective legislative act against
its detractors
Question 2
Select the sentence within the rst paragraph that
best exemplies the ultimate aim, according to the
passage, of the Gramm-Leach-Bliley Act.

Question 3
The author would consider which of the following
fallacies most similar to the fallacy perpetrated by
GBL?

 747s can y. 747s also have engines.


Therefore, those engines can also y.

 Any legislation that does not receive a


majority of votes will not be enacted. Thus
legislation that receives a majority of votes
will be enacted.
 Atoms are not visible to the naked eye.
Humans are made up of atoms. Therefore
humans may not always be visible to the
naked eye.
 Currently, no one has ever seen a purple
swan. Thus, no one will ever see a purple
swan.
 Savings accounts are a secure form of
investment, therefore everyone should
have a savings account.

LESSON 7 VERBAL

SCRATCH PAPER

The Princeton Review, Inc.

239

MANUAL FOR THE GRE

Line
5

10

7
15

20

25

30

35

40

45

50

240

The vermiform appendix is an unctuous deadend sac that hangs between the small and large
intestines. Some have concluded that the
appendix is the evolutionary remains of a larger
structure, the cecum, which was once used for
digesting food by ancestral organisms long-since
extinct. Some scientists, such as Charles Darwin
in On the Origin of Species (1859), classify the
appendix as a vestigial organ. However, such a
classication does not enjoy universal support. In
fact throughout medical history, several possible
purposes for the appendix have been offered,
evaluated, and refuted, including neuromuscular,
endocrine, and exocrine functions. A consensus
among medical specialists has begun to emerge,
speculating that the appendix serves some sort
of gastrointestinal immune function, citing the
organs concentrations of lymphoids and its
highly vascular structure as apparently homologous features between it and primate ceca. That
the appendix may help facilitate proper development of the immune system in young mammals
has been intimated by some studies on animals,
though not conclusively proven.
Utilizing cladistics, a new approach to evolutionary biology, which uses genetic information in
combination with a variety of other data to evaluate biological relationships that emerge over the
ages, Parker and his colleagues found supporting
evidence that further suggests that the appendix
may confer some evolutionary benet. That the
appendix has existed for at least 80 million years
and has evolved at least twice, once among Australian marsupials and later among rats, certain
other rodents, and particular primates is, by itself,
not compelling evidence that the appendix must
serve a function.
Admittedly the appendix contains a concentration of lymphoid tissue but such studies as those
conducted by Andersson and Buergal provide evidence that the appendix is actually maladaptive;
this lymphoid tissue seems prone to pathological inammatory states. Moreover, if the human
appendix were not vestigial, it would be as large
and developed as the cecum is in other primates
and would contribute signicantly to cellulose
fermentation.
However, evolutionary biology allows for a vestige to be a complex or specialized structureas
they are in the wings of an ostrich and the eyes
of the blind caveshyet still be a rudimentary
and degenerate relative to the same homologous
structures in other organisms. Those physicians

The Princeton Review, Inc.

55

who attempt to breach the domain of the evolutionary biologist and engage in phylogenetic
analysis of the appendix do so with a misguided
and long since abandoned assumption that evolutionary progress accompanies evolutionary
change.
Question 1
According to the author, which of the following is
true regarding the vermiform appendix?

 It aids the body in immunization.


 Its immunizing functions are rendered
useless by modern sanitation, giving it the
appearance of a vestigial organ.
 It is as large as primate ceca.
 It contributes signicantly to cellulose
fermentation.
 It is a vestigial organ.
Question 2
The author suggests that the reason medical
researchers believe that the vermiform appendix is
not vestigial is ultimately explained by

 recent studies that depict the immunizing


function of the appendix in humans
 incontrovertible evidence that
demonstrates immune function
 their correct application of cladistics
 the fact that doctors receive no training in
evolutionary biology and are limited solely
to expertise regarding human anatomy and
physiology
 partial ignorance on the part of medical
researchers regarding the eld of
evolutionary biology and its precise
taxonomy
Question 3
In the context, the breach mentioned on line 54
most clearly refers to

 the penetration of foreign bodies in an


organism
 the perforation of a ruptured appendix
 the misguided research conducted by
Parker and his colleagues
 the lack of communication between
evolutionary biologists and physicians
 the interdisciplinary research conducted by
doctors

LESSON 7 VERBAL

SCRATCH PAPER

The Princeton Review, Inc.

241

Lesson 8
Math

MANUAL FOR THE GRE

Distribution of Employees by Education and Gender in 2010


Highest Level Completed

Males

Females

High School

15

26

Undergraduate Degree

29

34

Graduate Degree

16

12

Doctorate

Total

65

80

Question 1

If the total number of employees in 2010 was 16


percent greater than it was in 2009, how many total
employees were there in 2009 ?

Question 2
For all positive integers a and b, where a 0 and
b 0,
1
a b =
1 1
+
a b
Quantity A

Quantity B

34

3
4

Question 3
If

15!

is an integer, what is the greatest possible


3m
value of m ?
4

244

The Princeton Review, Inc.

LESSON 8 MATH

SCRATCH PAPER

The Princeton Review, Inc.

245

MANUAL FOR THE GRE

Question 4
A

In the gure above, equilateral triangle BEG is


inscribed in square ACDF. The area of square ACDF
is 16.
Quantity A

Quantity B

The perimeter of the


shaded region

10

Question 5
2 x + 5y = 5
x 2y = 7
Quantity A

Quantity B

The value of x

The value of y

Question 6
Five students are participating in a relay race. The
rst and second students run for 1.2 miles each
at 5 miles per hour. The third student runs for 0.9
miles at 6 miles per hour. The fourth student runs
for 0.3 miles at 8 miles per hour. The fth student
runs for 0.4 miles at 10 miles per hour. Which of
the following is the average speed, rounded to the
nearest tenth, for all of the students?
5.4

246

The Princeton Review, Inc.

5.7

6.8

7.5

8.0

LESSON 8 MATH

SCRATCH PAPER

The Princeton Review, Inc.

247

MANUAL FOR THE GRE

Question 7
If a circle with a radius of 6 is graphed on a
coordinate plane such that the center of the circle is
at the origin, which of the following points would lie
outside the circle?
Indicate all such points.

.
.
.
.
.
.

(5, 1)
(3, 5)
(3, 4)
(4, 3)
(4, 5)
(5, 2)

Question 8
For a positive integer n, if the remainder is 1 when
2n is divided by 3, then which of the following must
be true ?
Indicate all such statements.

. n2 + 1 is a multiple of 3.
. 3n = (3)n
.

2n is an integer.

Question 9

z
O
z

The area of the circular region with center O


is 25.

248

The Princeton Review, Inc.

Quantity A

Quantity B

LESSON 8 MATH

SCRATCH PAPER

The Princeton Review, Inc.

249

MANUAL FOR THE GRE

Question 10
A sequence of numbers satises the equation
An = 2(An 1) + 1. If A4 = 10, what is the value of A1 ?

8
Question 11

In the parallelogram ABCD above, AD = 12. If the


area of ABE is

3
8

the area of parallelogram ABCD,

then EC =
3
2

8
3

Question 12
At the salad bar, Nicole can choose from 4 different
lettuce mixes, 8 different vegetables, 3 different
cheeses, and 5 different dressings. She must
choose one item from each category. She must also
decide whether or not to have croutons. How many
different combinations are possible?

250

The Princeton Review, Inc.

LESSON 8 MATH

SCRATCH PAPER

The Princeton Review, Inc.

251

MANUAL FOR THE GRE

DRILL
Question 1 of 8
2d + e + 2f = 0
3d e 5f = 0
For the above system of equations, if d 0, the ratio
of f to d is



 
 

3
2
5
3
2
3
3
5
5
3

Question 2 of 8
A scuba instructor sits on the edge of a square pool
with a side length of 8 meters, and a depth of 6
meters. 3 scuba divers are practicing in the pool.
Quantity A

Quantity B

The product of the


distances from the
scuba instructor to each
of the divers

1000 meters

Question 3 of 8
2
What is the value of 3
2
5

252

The Princeton Review, Inc.

x+
x+

2
9
2
15

y
?

LESSON 8 MATH

SCRATCH PAPER

The Princeton Review, Inc.

253

MANUAL FOR THE GRE

Question 4 of 8
In the sequence a, b, c, d, each number after the rst
is three times the number before it. If

d = a 18

what is the value of a ?

3 2

Question 5 of 8

C
2x

In the rectangle ABCD above, if AC = 20, what is the


area of the shaded region?

 50 2
 50 3
 100
 100 3
 200

Question 6 of 8
4j7
6k9
Which of the following could be the value of jk ?
Indicate all such values.

.
.
.
.
.
.

254

The Princeton Review, Inc.

10
13
16
24
48
90

9
2

LESSON 8 MATH

SCRATCH PAPER

The Princeton Review, Inc.

255

MANUAL FOR THE GRE

Advertising Budget for Company X by


Expenditure
Expenditure

Percentage of Budget

Television

52%

Internet

11%

Print

32%

Other

5%

Total in 2006: $210,000

Question 7 of 8

Of the money Company X spent on print advertising


1

was spent on newspaper advertisements


2
3
and
of the remainder was spent on yers.
4
Approximately how many thousands of dollars more
in 2006,

was spent on newspaper advertisements than on


yers?

 8.4
 13.6
 25.2
 33.6
 39.9

Question 8 of 8
Which of the following is NOT a factor of 1515 ?

 3
 9
 135
 180
 225

256

The Princeton Review, Inc.

LESSON 8 MATH

SCRATCH PAPER

The Princeton Review, Inc.

257

Lesson 8
Essays

MANUAL FOR THE GRE

Analytical Writing Instructions


Analyze an Issue
You will be given a brief quotation that states or implies an issue of general interest, and you will also be
given specific instructions on how to respond to that issue. Trained GRE readers will evaluate your response
according to how well you

260

respond to the specific task instructions


consider the complexities of the issue
organize, develop, and express your ideas
support your position with relevant reasons and/or examples
control the elements of standard written English

The Princeton Review, Inc.

LESSON 8 ESSAYS

SAMPLE ISSUE ESSAY PROMPTS


1.

Issue: One should not expect respect for disregarding the opinions of others. Only when every
point of view is taken into consideration should people take action.
Task: Write a response in which you discuss the extent to which you agree or disagree with
the statement and explain your reasoning for the position you take. In developing and
supporting your position, you should consider ways in which the statement might or might
not hold true and explain how these considerations shape yourposition.

2.

Issue: An idea alone, no matter how great, is meaningless unless it is put into practice.
Task: Write a response in which you discuss the extent to which you agree or disagree with
the claim. In developing and supporting your position, be sure to address the most compelling reasons or examples that could be used to challenge yourposition.

3.

Issue: A student who wishes to succeed in business should study anything but business while
in school. Once in the workplace, he or she will learn the skills of business, but the opportunity to gain additional perspectives by studying other fields of knowledge while in school is too
valuable to pass up.
Task: Write a response in which you discuss the extent to which you agree or disagree with
the recommendation and explain your reasoning for the position you take. In developing
and supporting your position, describe specific circumstances in which adopting the recommendation would or would not be advantageous and explain how these examples shape
yourposition.

4.

Issue: Unexamined conservatism is far more dangerous than reckless change.


Task: Discuss the extent to which you agree or disagree with the opinion stated above.
Support your views with reasons and/or examples from your own experience, observations,
or reading.

The Princeton Review, Inc.

261

MANUAL FOR THE GRE

Step 1: Brainstorm Examples


Heres a sample prompt.
Issue: To achieve great success, one must reject conventional thinking.

On your scratch paper draw a T that looks like this.

Success must be
unconventional

Success can be conventional

On one side write I agree and on the other write I disagree. Now take a moment to ask yourself, What
does it mean to agree or disagree with this prompt? Your essay prompt could come in one of three forms.
Here are some examples.
Extreme
No one can possibly achieve success in the world by conforming to conventional
practices and conventional ways of thinking.

Wishy-washy
People should treat experts with suspicion and mistrust because experts who have
specialized in a particular eld often have an overly narrow frame of reference.

Open-ended
The purpose of education is...

When your thinking runs dry, use this checklist:


Me, Friends, Family, School, City, Country, Company,
Species, The Very Old/Young, Science, History, Literature

262

The Princeton Review, Inc.

LESSON 8 ESSAYS

Step 2: Organize
Thesis: I believe ______________________________________________________________________
______________________________________because________________________________________
____________________________________ as seen through the examples of _____________________,
______________________, and _____________________.
Example 1: _________________________________ Notes: _________________________________

Example 2: _________________________________ Notes: _________________________________

Example 3: _________________________________ Notes: _________________________________

The Princeton Review, Inc.

263

MANUAL FOR THE GRE

Step 3: Write
When it comes to writing, there are two things that all essays must have, and then a laundry list of things
they could have to augment your score.

Must have:
Topic Sentences

Transitions

8
Could have:
Specics

Quotations

Commands

Analogies

Rhetorical Questions

Length

Big Words

264

The Princeton Review, Inc.

LESSON 8 ESSAYS

Practice

Issue: Unexamined conservatism is far more dangerous than reckless change.


Task: Discuss the extent to which you agree or disagree with the opinion stated above. Support your views
with reasons and/or examples from your own experience, observations, or reading.

Step 1: Brainstorm
8
Step 2: Organize
Thesis: I believe ______________________________________________________________________
___________________________________________ because __________________________________
__________________________________________ as seen through the examples of ______________,
______________, and ______________.
Example 1: _________________________________ Notes: _________________________________
Example 2: _________________________________ Notes: _________________________________
Example 3: _________________________________ Notes: _________________________________

Step 3: Write
First Paragraph: _____________________________________________________________________
__________________________________________________________________________________
__________________________________________________________________________________
__________________________________________________________________________________
__________________________________________________________________________________
__________________________________________________________________________________

The Princeton Review, Inc.

265

Verbal Practice

MANUAL FOR THE GRE

DRILL
Question 1
Accustomed to his wifes _________ nature, Stanton
was unfazed when she quickly changed her opinion
and just as quickly changed it back.
orid
painstaking
laconic
effusive
erratic

Question 2
The teens explanations, each more _________
than the last, only exacerbated his parents ire:
they considered every increasingly elaborate and
tortuous excuse a new offense.
opaque
conventional
lucid
byzantine
boorish

Question 3
One example that (i)________ the common
impression of the Dark Ages as a period of
intellectual stagnation is the development of
Carolingian miniature, the rst recognizably modern
handwriting. So (ii)________ was this new form
of writingthe most ancient surviving text of
nearly every classical literary work is in Carolingian
minisculethat Italian Renaissance scholars
assumed it was the original and called it Roman.

268

The Princeton Review, Inc.

Blank (i)

Blank (ii)

caricatures

intermittent

belies

equivocal

substantiates

ubiquitous

VERBAL PRACTICE

SCRATCH PAPER

The Princeton Review, Inc.

269

MANUAL FOR THE GRE

Question 4
Despite the recent (i)________ of ads that attack
political adversaries by name, many analysts have
begun to regard the strategy with a (ii)________
eye. Especially in crowded primaries, in which name
recognition is vital, they warn that using scarce
campaign resources to publicize your opponent
even in condemnationcould easily backre.

Blank (i)

Blank (ii)

paucity

dubious

novelty

convivial

proliferation

mercurial

Question 5
Economic policy is primarily reactionary, designed
solely to avoid downturns rather than foster upturns.
As a result, economic policies tend to be
(i)________ the previous crisis, rather than focusing
on (ii)________ management of the current boom.
Consequently, unrestrained nancial growth leads
to rampant speculation, and soon even seemingly
(iii)________ investments collapse as another
recession begins.
Blank (i)

Blank (ii)

Blank (iii)

ignorant of

prudent

solid

responses to

sparing

tenuous

peripheral to

irrational

unstable

Question 6
An increase in the cost of food staples would have
(i)________ consequences for the general population.
Although such an increase would (ii)________ the
need for the steep agricultural subsidies that the
government currently pays farmers, it is unlikely that
the savings would be passed down to the taxpayers
to help (iii)________ increased food expenditures.

270

The Princeton Review, Inc.

Blank (i)

Blank (ii)

Blank (iii)

unexpected

squander

augment

adverse

abate

offset

advantageous

drive

ag

VERBAL PRACTICE

SCRATCH PAPER

The Princeton Review, Inc.

271

MANUAL FOR THE GRE

Line
5

10

15

9
20

Human-to-human blood transfusions in the


nineteenth century were plagued by two major problems: rst, the necessity of transfusing
blood as soon as it was drawn, due to coagulation, and second, the occasionally fatal hemolytic
reactions, often accompanied by fever or nausea,
that aficted many patients. Rudimentary anticoagulants were developed as early as 1860, solving the rst obstacle, and Landsteiners classication of ABO blood groups in 1900 was presumed
capable of concluding the second. However, even
after eliminating disbursement of mismatched
blood due to clerical error, a small percentage
of recipients still experienced symptoms that
paralleled those of a hemolytic reaction. Later
advances in blood analysis revealed not only that
there are numerous additional blood groups with
which donors may be incompatible, notably the
Rh system, but also that white blood cells may
attack new blood from even seemingly suitable
donors.
Question 7
The author mentions the difculties associated with
blood transfusions in order to
criticize the simplicity of early attempts at
transfusions
 present supplemental information that
could be used to help reduce these
difculties
 suggest that it will never be possible to
understand all the causes of a hemolytic
reaction
 argue for increased measures of
compatibility across various blood groups
 describe best practices for blood
transfusions in order to prevent the
difculties

272

The Princeton Review, Inc.

Question 8
The passage suggests that the author believes
which of the following about transfusions?

 ABO and Rh matching, when used together,


are sufcient to ensure a successful
transfusion.
 White blood cells are classied according
to different classications than are the
other components of blood.
 Before classications were understood, the
majority of blood transfusion recipients
died.
 Accurate blood group matching during
transfusions will eliminate the chance of
having a hemolytic reaction, or symptoms
similar to one.
 Donor-to-recipient blood compatibility
depends more on some blood groups than
others.
Question 9
The primary purpose of the passage is to
highlight two major problems with human
blood transfusions
describe the origins of human blood
transfusions
examine the difculties associated with
transfusing blood
recommend improvements to human
blood transfusion methods
illustrate the problems presented by blood
type in human blood transfusions

VERBAL PRACTICE

SCRATCH PAPER

The Princeton Review, Inc.

273

MANUAL FOR THE GRE

Line
5

10

15

9
20

25

30

35

40

45

50

274

Despite his unquestioned preeminence in


Rome from 31 BCE to 14 CE, Augustus took pains
to present himself not as king, dictator, or
emperor, but as princepsrst among equals.
The exact nature of Augustuss authority has long
been a subject of debate among historians. In
particular, much attention has been paid to the
role played by Augustuss provincial reform in
both consolidating his power and furthering his
public image as a champion of traditional republican values.
The most visible element of the provincial
reform, carried out between 27 and 23 BCE, was
the division of the Empire into two types of
provinces: imperial and Senatorial. In imperial
provinces, which included such troublesome frontier areas as Spain, Gaul, and Syria, Augustus
retained the highest ofcial form of power,
proconsular imperium. Senatorial provinces,
conversely, were governed by proconsuls chosen
by the Senate, thus reecting the time-honored
republican belief in the balance of power. In addition to restoring prestige to the Senate as a body,
this development also opened up one of the
primary ofces by which Roman aristocrats had
traditionally achieved status, as did Augustuss
resignation of the Roman consulship in 23 BCE,
ending his decade-long monopolization of one of
the Republics highest ofces.
No historian has suggested an equal division
of power between Augustus and the Senatethe
frontier nature of the imperial provinces meant
that the provinces held most of the armybut
this division was widely held to be real by modern observers. In the words of Mommsen, the
greatest classicist of the nineteenth century,
Augustus had created a dyarchy, or joint rule
between princeps and Senate. This vision dominated accounts of Augustuss rule until the discovery of a series of inscriptions in North Africa
in the early 1920s.
These inscriptions, which came to be known
as the Cyrene Edicts, date to around 6 BCE, more
than a decade after Creta et Cyrenaica was created as a Senatorial province. Nonetheless, they
show Augustus intervening in such local issues
as the empanelling of jurors. The degree to
which the discovery of the Cyrene Edicts shattered the historical orthodoxy is best seen in
Symes epoch-making 1974 work, The Roman
Revolution. Syme contends that Augustus was, in
fact, granted proconsular imperium over the
entire Empire, and that Senatorial proconsuls
were effectively his legates.

The Princeton Review, Inc.

Question 10
The passage suggests which of the following about
traditional republican values in the era of Augustus?

 Roman nobles could only achieve status


through ofce-holding.
 An autonomous Senate was considered a
signicant element.
 Traditional republican values allowed the
Emperor to have proconsular imperium
over the entire Empire.
 The army should be evenly distributed
throughout the Empire.
 Traditional republican values were rooted in
the dual mythos of the farmer-soldier.
Question 11
Which of the following questions can be answered
by information provided in the passage?

 Why did Augustus resign the Roman


consulship in 23 BCE?
 What accounts for the preservation of the
Cyrene Edicts?
 What were some of the ofces that
affected how Roman society judged its
elites?
 Was Augustus the rst to present himself
as princeps?
 Were there more imperial or Senatorial
provinces?
Question 12
According to the passage, the Cyrene Edicts are
important because they

 prevented Senators from being empanelled


as jurors in imperial provinces
 suggested that Symes views on imperial
authority were too extreme
 conclusively refuted Mommsens
assessment of provincial reorganization
 authorized Augustus to reorganize the
Roman provinces
 cast doubt on the clear distinction between
Senatorial and imperial provinces

VERBAL PRACTICE

SCRATCH PAPER

The Princeton Review, Inc.

275

MANUAL FOR THE GRE

Question 13

Question 16

Every new scientic theory that challenges the


reigning orthodoxy is viewed as _________ until it
is supported by mounting evidence and eventually
adopted as truth.

Audiences were rapt during the rst half of the


movie; it was only after the jejune plot twist that
their interest began to _________.

.
.
.
.
.
.

dichotomous
suspect
heretical
middling
critical
inconsequential

.
.
.
.
.
.

ag
wax
swell
prevaricate
converge
ebb

Question 17
Question 14

The concepts of please and thank you are usually


taught together in the same lesson when we are
young, which makes it ironic that the former
is _________ used while the latter is quite commonly
heard and read in daily communication.

.
.
.
.
.
.

oft
rarely
frequently
seldom
redundantly
signicantly

Question 15
Although our current permissive society allows more
explicit expressions, the tradition of _________ lives
on.

.
.
.
.
.
.

276

rhetoric
euphemism
excoriation
garrulousness
genteelism
pragmatism

The Princeton Review, Inc.

As evidence of the severe damage inicted on the


American automobile industry by competition from
foreign automakers, an expert pointed to a 27%
drop in sales of American-made cars in the state of
California over the last year.
Which of the following, if true, most seriously
weakens the argument that the American
automobile industry has been weakened by foreign
competition?

 Only 68% of Californians over the age of


16 own cars.

 Fewer than one-third of California drivers


report having negative feelings about
American-made cars.
 Major improvements in public
transportation were made in California
eighteen months ago.
 Over the last year, foreign auto companies
have signicantly increased their
advertising budgets for American television
commercials.
 Total car sales in the state of California did
not signicantly decrease over the last year.

VERBAL PRACTICE

SCRATCH PAPER

The Princeton Review, Inc.

277

MANUAL FOR THE GRE

Line
5

10

15

9
20

25

30

Until about 1870, the eld of linguistics was


marked by three successive approaches. Language was studied rst through grammar, which
provided a set of rules for a particular language,
then through philology, which sought to discover truths about the customs and traditions
of ancient cultures through language, and nally
through comparative linguistics, which posited
a series of afnities between seemingly diverse
languages.
For Ferdinand de Saussure, such linguistic paradigms were unsatisfactory. Saussure theorized
that the constructs of language are inherently
arbitrary, and that to truly understand linguistics
one needed to retreat from the notion that the
evolution of words themselves was signicant.
As an example, he suggested that there is no
real relationship between the word tree and
the concept that people associate with that word.
The word derives its meaning purely from the
fact that speakers use it as a sign for a mutually
agreed upon concept. One objection Saussure
anticipated to his theory was the existence of
onomatopoeia, commonly dened as language
that sounds like its meaning (bam, squish,
etc.). Saussure dismissed this objection by suggesting that true onomatopoeia, as opposed to
that produced by conscious attempts at
onomatopoeic pronunciation, is both rare and to
some extent socially constructed.
Question 18
Saussures dismissal of onomatopoeia as a
challenge to his theory would be most strengthened
if he did which of the following?

 Showed that conscious attempts at


onomatopoeic pronunciation were socially
constructed
 Offered an example of an onomatopoetic
word that emerged independently in
several languages
 Challenged the validity of grammar,
philology, and comparative linguistics as
linguistic disciplines
 Demonstrated that not all linguists agree
on the frequency with which onomatopoeic
words occur in language
 Provided an example of a particular word
generally regarded as onomatopoeic and
gave a detailed explanation of why it is not
true onomatopoeia

278

The Princeton Review, Inc.

Question 19
In which sentence does the author present a set of
methodologies that Saussure nds inadequate?

Question 20
Responding to complaints from administrative
assistants about back pain after long periods of
sedentary assignments, an ofce commissioned a
study to determine whether sitting for long periods
of time induces back pain. Researchers asked the
assistants how often they completed assignments
requiring long periods of sitting, and how often
they had experienced back pain during the past six
months. The study found that the assistants who
spent the most time engaged in sedentary activities
were most likely to report frequent back pain. The
study thus concluded that sitting for long periods of
time does indeed cause back pain.
Which of the following, if true, would most
undermine the argument?

 A third question in the survey asked


whether respondents engaged in a regular
exercise regimen, and they all said that
they did not.
 The researchers did not ask the
administrative assistants whether the back
pain they experienced was severe or mild.
 Another recent study has shown that
sitting for long periods of time can cause
leg pain.
 Administrative assistants who said that
they experienced frequent back pain were
especially likely to over-report the amount
of time they spent sitting down.
 Administrative assistants who reported
frequent sedentary activity were no more
dissatised with their jobs than were those
who did not report frequent sedentary
activity.

VERBAL PRACTICE

SCRATCH PAPER

The Princeton Review, Inc.

279

Math Practice

TRIGGER

Find the 45-45-90 triangle


(you know its in there somewhere).

RESPONSE

TRIGGER

Right triangle with 30, 60,


or 3 , a big side that is twice
the small side, or height of
equilateral triangle

Find the 30-60-90 triangle


(you know its in there somewhere).

RESPONSE

Arc, sector, or a fraction of a


circle

Write your formula:


angle
360F

arc
area sector
=
circumference area circle

TRIGGER
TRIGGER

Overlapping shapes

Find the elements the two


shapes have in common.

TRIGGER

Geometry on Quant Comp or


Must Be questions

Draw the shape twice. Stretch


and squash.

RESPONSE

Shaded regions or strangelooking shapes

RESPONSE

282

RESPONSE

Find familiar shapes.

RESPONSE

Right triangle with 45 or 2 ,


or diagonal of a square

TRIGGER

MANUAL FOR THE GRE

The Princeton Review, Inc.

Shaded = Total Unshaded


Draw lines (split, connect, draw
radii, drop heights, or otherwise
play with the shape).

percent change
percent decrease/less

difference
100
original

Draw a pie every time the word


average appears in the question.

median

Find a bunch of numbers and list


them in order on your scratch
paper.

rate

Exponents

Write formula: D = R T

Factor, regroup, try to cancel


out common factors.

RESPONSE

TRIGGER

speed

RESPONSE

TRIGGER

TRIGGER

average

RESPONSE

Draw ratio box on scratch paper.

RESPONSE

ratio

RESPONSE

TRIGGER

Percent change =

TRIGGER

percent increase/more

Write formula:

RESPONSE

TRIGGER

MATH PRACTICE

The Princeton Review, Inc.

283

TRIGGER

Convert large bases to their


prime numbers.

The phrases how many,


how much, what is the value of

1. List answers on your scratch paper.

or

3. Assume (C) to be correct.

An urge to write your own


algebraic formula

4. Use (C) to work the problem.

2. Label the rst column.

5. POE.

If fractions, plug in common


denominator.
If percentages, plug in 100 for total.

RESPONSE

Plug In for the unknown value or


amount.
Answer choices expressed as
fractions or percentages

RESPONSE

TRIGGER

Numbers too big to calculate

RESPONSE

TRIGGER

MANUAL FOR THE GRE

2. Replace variable with number.


Variables in the answer
choices

3. Work the problem.


4. Identify and circle target number.

RESPONSE

TRIGGER

1. List A,B,C,D,E on scratch paper.

TRIGGER
|

The Princeton Review, Inc.

Quant Comp with variables

2. Plug in an easy number.


3. Eliminate two answer choices.
4. Repeat using FROZEN.

1. Draw set-up.
Must be

2. Plug in an easy number.


3. Eliminate answer choices.
4. Repeat using FROZEN.

RESPONSE

284

1. Draw set-up.

RESPONSE

TRIGGER

5. Check all answer choices.

TRIGGER
TRIGGER

Fill in the percentages.

l1 || l2

Find and label your big and small


angles.

Parallel lines

Write area formula on scratch.


...area of triangle...

...area of rectangle...
...area of parallelogram...

Plug in information directly


underneath.

Draw the shape, label the sides,


and add the sides together.

Write r d C A vertically on
scratch paper.
Find radius, diameter,
circumference, and area.

RESPONSE

Circles

Write the area formula on


scratch.

RESPONSE

...perimeter...

Plug information directly


underneath.

RESPONSE

TRIGGER

...standard deviation...

RESPONSE

TRIGGER

Draw the curve.

RESPONSE

TRIGGER

...normal distribution...

RESPONSE

TRIGGER

MATH PRACTICE

The Princeton Review, Inc.

285

MANUAL FOR THE GRE

45-45-90
Right triangles

30-60-90
Pythagorean triples

RESPONSE

TRIGGER

Look for:

TRIGGER
TRIGGER
TRIGGER
TRIGGER

Draw Group Table.

Put the equation in the form


x + bx + c = 0
Underneath the equation, write
down (x )(x ) = 0
Remember A.M.
If necessary, nd every factor of c.

286

The Princeton Review, Inc.

RESPONSE

x and x 2 in the same equation,


or anything that looks like a
quadratic

Total = [Group 1] + [Group 2] [Both] +


[Neither]

RESPONSE

Group question in which


elements are in either one
group or other (no overlap)

Write down group formula:

RESPONSE

Group problem with overlap,


both, or neither

Find the probability an event


wont happen and subtract
from 1.

RESPONSE

Probability question asks not


or at least

Look to factor or expand.

RESPONSE

Factorials

TRIGGER

(3:4:5, 6:8:10, 5:12:13)

Question asks for the distance


between two points

Draw triangles and look for


Pythagorean triples.

...slope...

vertical change y Rise y 2 y 1


=
=
=
horizontal change x Run x x
2

TRIGGER
TRIGGER
TRIGGER

Interior diagonal of a box

Draw a space for each event.


want

Find the total(s). total

a2 + b2 + c2 = d 2

RESPONSE

...probability...

Park the information you have


directly underneath.

RESPONSE

...on a coordinate plane...

RESPONSE

Write y = mx + b on scratch paper.

RESPONSE

TRIGGER

RESPONSE

TRIGGER

MATH PRACTICE

The Princeton Review, Inc.

287

MANUAL FOR THE GRE

DRILL
Question 1

Question 4

40 percent of a certain number is 1,992. What is


95 percent of that number?

Question 2
n is an even integer, such that 5 < n < 15.

k is an integer.
Quantity A
5

Quantity B
5

5 + 3k

5 + 7k

 Quantity A is greater.
 Quantity B is greater.
 The two quantities are equal.
 The relationship cannot be determined
from the information given.

Quantity A

Quantity B

(3)n

(3n)

 Quantity A is greater.
 Quantity B is greater.
 The two quantities are equal.
 The relationship cannot be determined
from the information given.
Question 3
Miki learns that b books cost d dollars. At this rate,
how much will Miki pay for seven books?

7b
d

 7db


288

b
7d

7d

b
7b

The Princeton Review, Inc.

Question 5
A group of 5 coworkers is going to equally divide
the proceeds, after taxes, of a winning lottery ticket.
If taxes consume 40% of the total prize, and each
coworker will receive $1,800,000, what is the total
amount of the prize?

 $15,000,000
 $17,500,000
 $20,000,000
 $22,500,000
 $25,000,000

MATH PRACTICE

SCRATCH PAPER

The Princeton Review, Inc.

289

MANUAL FOR THE GRE

Question 6

Question 8

If 0 < x < 1, which of the following must be greater


than x ?

In a certain museum exhibit,

Indicate all such values.

.
.
.
.
.
.

x
2x
1 x2
1 2x
x2 2
2 x2

Question 7
Of the 20 water heaters for sale at AAA Appliances,
half are both Energy Star certied and eligible for a
federal tax rebate. If 14 water heaters are Energy
Star certied, and the number of water heaters
eligible for the federal tax rebate is 3 times the
number that are neither Energy Star certied nor
eligible for the federal tax rebate, then how many
of the water heaters are eligible for the federal tax
rebate?

 4
 12
 14
 15
 18

of the artifacts are


5
Paleolithic, and the remaining 16 are Neolithic. Of
the Paleolithic artifacts,

are Mediterranean. If
4
26 of the artifacts are Mediterranean, how many
Neolithic artifacts are NOT Mediterranean?

Question 9
B

135
A

Which of the following could be an angle


measurement within parallelogram ABCD above?
Indicate all such values.

.
.
.
.
.
.
.

45
55
80
100
115
135
225

Question 10
B
105

30

If the length of BC is 6 2, then the area of triangle


ABC =

 18
 18 3
 36
 18 + 18 3
 36 + 36 3
290

The Princeton Review, Inc.

MATH PRACTICE

SCRATCH PAPER

The Princeton Review, Inc.

291

MANUAL FOR THE GRE

Question 14

Question 11

y
C

R (20,5)
A
Note: Figure not drawn to scale

x
S

In the gure above, if BC = 8 and CD = 4 2 , what


is the area of the parallelogram ABCD ?

 16
 32
 32 2
 32 3
 64

The equation of the line in the rectangular coordinate


5
system above is y = x + 10. What is the distance
4
from point S to point R ?

Question 12
36

. What is the

circumference of Circle O ?

Circle O has an area of

5
 5 5
 12
 13
5 17
Question 15

Question 13

What is the value of the expression

12! + 11!
10!

 2.3
 23!
 13!
 143
 264

s
If the area of the circular region above is 4, what is
the area of the square with sides of length s ?

4
 2
 22
8
 16

292

The Princeton Review, Inc.

MATH PRACTICE

SCRATCH PAPER

The Princeton Review, Inc.

293

MANUAL FOR THE GRE

Question 16

Question 18

In a study of 130,000 teenagers, the number of


hours of television watched per teenager in the
month of October had a normal distribution. If the
average number of hours watched was 90 and
the standard deviation was 5 hours, which of the
following must be true?

Three students, Mark, Peter, and Wanda, are


all working on the same math problem. If their
1 2
individual probabilities of success are , , and
4 5
3
, respectively, then what is the probability that
8
at lease one of the students will get the problem

Indicate all such statements.


correct?

. 18,200 teenagers watched between 95 and


.
.

100 hours of television in October.


More than 25% of the teenagers watched
between 80 and 85 hours of television in
October.
2600 teenagers watched fewer than 80
hours of television in October.

Question 17




partygoers will be chosen by lottery to win a prize.


1
1
If of the male partygoers and of the female
5
2
partygoers arrive late, what is the probability that the
prize will be won by a partygoer who arrives late?

80
9
32

23

77

Colleen is hosting a party for 125 people, 75 of


whom are male. At the end of the party, one of the

32
80
39
40

Question 19
A grandmother wants to display photos of her
grandchildren on her mantel. She has one photo of
each of her 11 grandchildren, but she can t only
four photos on the mantel. How many different
arrangements of photos are possible?


330

484

7,920
 1,663,200
 9,979,200
Question 20
There are nine high school basketball teams in
Johnston County. Each team plays each of the
other teams in the county exactly once. In addition,
each team plays two games against schools from
a neighboring county. How many total games are
played?

 36
 54
 90
 108
 126
294

The Princeton Review, Inc.

MATH PRACTICE

SCRATCH PAPER

The Princeton Review, Inc.

295

Between Now and


Test Day

MANUAL FOR THE GRE

PICK A SUBJECT AND MASTER IT


There are only about 20 concepts tested on the whole GRE. Some of these you are comfortable with and
some you are not. Between now and test day, pick one or two distinct topics you havent yet mastered and
master them.

298

The Princeton Review, Inc.

BETWEEN NOW AND TEST DAY

HI

AS

TE
GH R E D
L O PRI
W OR
PR I T
IO Y
RI
TY

Verbal

Vocab
Text Completion Strategy
Sentence Equivalence Strategy
Reading Comprehension Passage Profiling
Reading Comprehension Question Strategy
Argument Question Strategy

HI

AS

TE

GH R E D
L O PRI
W OR
PR I T
IO Y
RI
TY

Math

Fundamentals
Plugging In
Plugging In the Answers (PITA)
Quantitative Comparison/Must Be
Fractions/Decimals/Percentages
Ratios and Proportions
Exponents and Square Roots
Mean, Median, Mode, and Statistics
Parallel Lines
Triangles
Quadrilaterals
Circles
3D Figures
Coordinate Geometry
Charts, Graphs
Linear Equations and Inequalities
Quadratic Equations
Probability
Rates
Groups, Sequences, and Functions
Factorials
Arrangements and Combinations

HI

AS

TE

GH R E D
L O PRI
W OR
PR I T
IO Y
RI
TY

Essays

Issue Essay
Argument Essay
The Princeton Review, Inc.

299

MANUAL FOR THE GRE

A FEW LAST THINGS TO NOTE


Practice Tests and Drills
Once you have taken them all, delete them all and do them again. The second time around, the score is
moot because you will have seen many of the questions. Thats okay. Practice your technique and use your
scratch paper.

The First Run Is a Practice Run


If possible, plan to take the GRE more than once. This allows you to relax the first time. If you get a great
score, you are done. If you need more points, youve got a second option.

Dont Start Cold


Bring your warm-up page and do it before the test.

Stick to the Plan


Its easy to get stressed out on test day, so focus on the positive: You have a plan. Youve applied it on the
practice tests. Stick to it. Go slowly. Focus on accuracy. Do the easy questions first. If you get stuck on a
problem, mark it and come back.

Standardized = Predictable

You have now seen every subject and question type you could see on test day. There should be no surprises.
Most of what you see will look very familiar.

300

The Princeton Review, Inc.

BETWEEN NOW AND TEST DAY

Arithmetic
0:
1:
2:

Exponents

Even integer. Not prime. Not positive. Not


negative
Odd positive integer. Not prime
Even prime integer. First prime
Primes less than 20: 2, 3, 5, 7, 11, 13, 17, 19

Multiply
Add
Divide
Subtract
Power
Multiply

Order of Operations (PEMDAS):

x1 = x
x0 = 1

Parentheses
E xponents

1x = 1
0x = 0
(x)even = positive
(x)odd = negative

Multiplication & Division


Addition & Subtraction

Percents

Quadratics

%:

100

x 2 + bx + c = 0 (x + d)(x + e) = 0

of:

(times)

is, are, was:

what:

x, y, z (variable)

(x y)2 = x 2 2xy + y2

Percent Change =

difference
original

(x + y)(x y) = x 2 y2

d+e=b

de = c

(x + y)2 = x 2 + 2xy + y2

Rate

Groups
Total = Group 1 + Group 2 + Neither Both

Probability

Distance = Rate Time


Work = Rate Time
Avg Speed = Total Distance Total Time

Average
0

want
1
total

Probability of events A and B = A B

Total
# of Average
things

Probability of events A or B = A + B
Probability of at least once = 1 probability of never
The Princeton Review, Inc.

301

MANUAL FOR THE GRE

Triangles

Quadrilaterals

Angles add up to 180

Angles add up to 360


Perimeter: add up the sides

1
A = bh
2

Area of parallelogram (rectangle or square) = b h

Isosceles triangle: Two equal sides are opposite two


equal angles.
Similar triangles: If all angles of two triangles are the
same, then corresponding sides are proportional.

Circles
d = 2r
C = d = 2r
A = r 2

Third Side Rule


Any side of a triangle is greater than the difference of
the other two sides, and smaller than the sum.

Length of Arc =

angle
Circumference
360

Right Triangles

Area of Section =

angle
Area of Circle
360 

a +b =c
2

3 : 4 : 5;

6 : 8 : 10;

5 : 12 : 13

a : a : a 2 (45:45:90)

Coordinate Geometry
Line Formula: y = mx + b (m is slope. b is y-intercept)

45
a 2

Slope:

45
a

rise y 2 y1
=
run x 2 x1

Solids
Volume of Cube: V = s3

a : a 3 : 2a (30:60:90)

Volume of Box: V = l w h
Volume of Cylinder: V = r 2h
30
2a

a 3
a

302

The Princeton Review, Inc.

60

Diagonal of Box: a2 + b2 + c 2 = d2

NOTES

NOTES

NOTES

NOTES

Das könnte Ihnen auch gefallen